Practice MBE Set

अब Quizwiz के साथ अपने होमवर्क और परीक्षाओं को एस करें!

IN an automobile collision case brought by a plaintiff against a defendant, the plaintiff introduced evidence that a bystander made an excited utterance that the defendant ran the red light. The defendant called a witness to testify that later the bystander, now deceased, told the witness that the defendant went through a yellow light. The witness's testimony should be:

A. Excluded, because it is hearsay not within any exception. B. Excluded, because the bystander is not available to explain or deny the inconsistency. C. Admitted only for the purpose of impeaching the bystander. D. Admitted as impeachment and as substantive evidence of the color of the light. Answer: C. When a hearsay statement has been admitted in evidence, the credibility of the declarant may be attacked, and if attacked may be supported by any evidence that would be admissible if declarant had testified as a witness.

A catering company entered into a written contract with a dish supplier to purchase 5,000 plastic dishes at $.10 per dish. The contract called for the supplier to deliver the 5,000 dishes to the caterer on or before October 1. On October 1, the supplier delivered only 3,000 dishes to the caterer. The supplier informed the caterer that it was experiencing manufacturing delays and would deliver the other 2,000 dishes by October 31 at the latest. The caterer accepted the delivery of the 3,000 dishes, but because it had a number of catering jobs lined up for early October, the caterer was forced to purchase 2,000 dishes from another supplier at a price of $.12 per dish. The supplier demanded that the caterer pay $300 for the 3,000 dishes delivered, but the caterer refused to pay anything. If the supplier sues the caterer for breach of contract, what will the supplier recover?

A. $300, the price under the contract for the 3,000 dishes that were delivered, with no deduction for the caterer's extra cost, because the caterer waived its right to cover when it accepted the supplier's tender without expressly reserving its rights. B. The reasonable value of the $3,000 dishes that were delivered less $40, which is the extra cost incurred by the caterer to obtain the balance of the dishes. C. $260, the price under the contract for the 3,000 dishes that were delivered less $40, which is the extra cost incurred by the caterer to obtain the balance of the dishes. D. Nothing, because the supplier is in material breach of the contract until it tenders delivery of the last 2,000 dishes. Answer: C. The supplier will recover $260. Dishes are moveable goods, and so UCC applies. When goods fail in any respect to conform to the contract, the buyer may accept the goods, and pay the contract price for the goods accepted. However, the buyer has a right to offset its damages. When a seller fails to deliver goods as promised, the buyer may "cover" under UCC by making a reasonable purchase of substitute goods, and then may recover as damages the difference between the contract price and the "cover" price.

Two friends planned to incorporate a business together and agreed that they would own all of the corporation's stock in equal proportion. A businesswoman conveyed land by a warranty deed to "the corporation and its successors and assigns." The deed was recorded. Thereafter, the friends had a disagreement. No papers were ever filed to incorporate the business. There is no applicable statute. Who owns the land?

A. The businesswoman, because the deed was a warranty deed. B. The businesswoman, because the deed was void. C. The two friends as tenants in common, because they intended to own the corporation's stock in equal proportion. D. The two friends as tenants in common, because they were the intended sole shareholders. Answer: B. To be valid, a deed must be properly executed and delivered. A deed to a nonexistent grantee, such as a corporation that has not yet been legally formed, is void. At the time the businesswoman attempted to convey the land to the corporation, the corporation had not yet been legally formed, so the deed was void.

A geologist filed a diversity action against a farmer in federal court in State A, seeking recovery for the damage to her very expensive sports car and the broken arm that she suffered as a result of a car crash with the farmer. At trial, after two days of testimony, the jury returned a verdict for the geologist and awarded her $75,000 in compensatory damages. Under federal law, a judge who believes compensatory damages are so excessive as to "shock the conscience" can offer the plaintiff the choice between a new trial or remittitur of the excessive damages. Under substantive State A law, the court can reduce a jury's awards by way of remittitur if it thinks damages are "excessive." Which of the following is true?

A. The farmer should submit only a motion for remittitur, on the ground that the jury's damage award "shocks the conscience." B. The farmer should submit a joint motion for new trial and remittitur, on the ground that the jury's damage award "shocks the conscience." C. The farmer should submit only a motion for remittitur, on the ground that the jury's damage award was "excessive." D. The farmer should submit a joint motion for new trial and remittitur, on the ground that the jury's damage award was excessive." Answer: D. Under the Erie doctrine, federal courts sitting in diversity apply the remittitur rules of the state in which they are situated; hence the appropriate remittitur standard is "excessive" rather than "shocks the conscience." **The courts must offer plaintiff the choice between a new trial or the reduced award. A motion for remittitur accordingly must be paired with a motion for a new trial.

In 2000, the owner of a 100-acre tract prepared and duly recorded a subdivision plan called Happy Acres. The plan showed 90 one-acre lots and a ten-acre tract in the center that was designated "Future Public School." The owner published and distributed a brochure promoting Happy Acres which emphasized the proximity of the lots to the school property and indicated potential tax savings "because the school district will not have to expend tax money to acquire this property." There is no specific statute concerning the dedication of school sites. The owner sold 50 of the lots to individual purchasers. Each deed referred to the recorded plan and also contained the following clause: "No mobile home shall be erected on any lot within Happy Acres." A woman was one of the original purchasers from the owner. In 2006, the owner sold the remaining 40 lots and the ten-acre tract to a man by a deed which referred to the plan and contained the restriction relating to mobile homes. The man sold the 40 lots to individual purchasers and the ten-acre tract to a friend. None of the deeds from the man referred to the plan or contained any reference to mobile homes. In 2007, the school board of the district in which Happy Acres is situated has voted to erect a new school on the ten-acre tract. In an appropriate action between the school board and the friend to determine title, the result will be in favor of

A. the friend, because the school board has been guilty of laches. B. the friend, because his deed did not refer to the subdivision plan. C. the school board, because the friend had constructive notice of the proposed use of the tract. D. the school board, because there has been a dedication and acceptance of the tract. Answer: D. The issue being tested here is the dedication of land for public use which requires two elements: (1) dedication, and (2) acceptance. Under common law, an offer of dedication may be made by: (1) a written or oral statement, (2) the submission of a map or plat showing the dedication, or (3) the opening of the land to public use. Acceptance can occur by: (1) a formal resolution, (2) the approval of the map or plat, or (3) the actual assumption of maintenance or construction of improvements. Here the dedication took place when the owner recorded the plan of the subdivision, marking the land to be set aside for the future school. Once land has been dedicated, it can be accepted at anytime unless the dedication is revoked by the original grantor. Additionally, the public cannot lose dedicated lands by non-use or delay in use of the lands for the purpose for which they were dedicated. The conveyance from the owner to the man did not revoke the dedication because the man was aware of the reservation. Therefore, the school board was still able to accept the dedication when it voted in 2007. The conveyance from the man to the friend also did not revoke the dedication. However, the friend will likely be able to recover from the man the value of the property lost plus the cost of removing any improvements made to the land because the man was responsible for the friend's ignorance of the reservation of the land for public use.

A plaintiff sustained personal injuries in a three-car collision caused by the concurrent negligence of all three drivers. In the plaintiff's action for damages against the other two drivers, the jury apportioned the negligence 30% to the plaintiff, 30% to driver #1, and 40% to driver #2. The plaintiff's total damages were $100,000. Assume that the state has retained the common law rule pertaining to contribution and that the state's comparative negligence statute provides for a system of pure comparative negligence but abolishes joint and several liability. If the plaintiff chooses to pursue the claim against driver #1 alone, she will be entitled to collect at most

A. $70,000 from driver #1, and then driver #1 will be entitled to collect $40,000 from driver #2. B. $30,000 from driver #1, and then driver #1 will be entitled to collect $10,000 from driver #2. C. $30,000 from driver #1, and then driver #1 will be entitled to collect nothing from driver #2. D. Nothing from driver #1, because his percentage of fault is not greater than that of the plaintiff. Answer: C. The facts tell you that joint and several liability has been abolished, which means that each defendant is liable only for his own share of the damages, not the entire award. In addition, contribution between tortfeasors is only available where one defendant has paid more than his determined share of the damages.

A large producer of breach wrote to a distributor of flour, asking, "How much will you charge to supply my needs for flour for the next year?" The distributor replied in writing that it could supply the producer with all the flour it would need next year at a specified price per pound. The producer wrote back, "Your offer to supply me with flour is hereby accepted, provided that you agree to a 10% discount if payment is made within 10 days from date of billing." What should the producer's reply concerning a 10% discount be characterized as?

A. An acceptance with a proposal for an additional term. B. An acceptance with a term that, absent an objection, is part of the contract. C. A proposed modification of the contract. D. A rejection of the distributor's offer. Answer: D. The producer's reply is a conditional acceptance, which is a rejection of the offer. This question deals with the "battle of the forms" provision of the UCC. Under section 2-207 of the UCC, an acceptance containing additional or different terms is effective as between merchants unless the offeree expressly makes his acceptance conditional on assent by the offeror to the additional terms. When acceptance is made expressly conditional on the acceptance of new terms, it is a rejection of the offer. The conditional acceptance is essentially a new offer, and the original offeror may form a contract by expressly assenting to the new terms.

An uncle who owned a large tract of wooded land in fee simple had always opened his land to hunters from the local hunting club, and had often expressed the wish that they be permitted to continue to use it after he died. On his 75th birthday, he conveyed the land to his niece and nephew, who were members of the hunting club. The deed was a general warranty deed conveying the property in fee simple absolute. A few days later, the niece had a serious dispute with the nephew and the other members of the hunting club, and she quit the club. The uncle then executed a written agreement with the nephew stating that the conveyance of the land to the niece and nephew was in trust for the benefit of the local hunting club for a period of 10 years, with the niece and nephew in fee simple. Several months later, the uncle died. When the next hunting season drew near, the nephew told the niece that members of the hunting club were once again planning to hunt on the property. The niece threatened to have anyone hunting on the property other than the nephew arrested for trespassing. Legal or equitable relief to establish his rights and the rights of the hunting club. What, if any, relief should the court provide?

A. Deny the nephew any relief, because the niece has done nothing that would constitute an ouster of the nephew. B. Partition the land into two separate tracts so that the nephew may permit the hunting club to use his half. C. Order the niece to permit the hunting club to hunt on the land, because the uncle created an inter vivos trust with the requisite formalities for the benefit of the hunting club. D. Order the niece to permit the hunting club to hunt of the land, because the nephew is equally entitled to possession of all of it and can allow members of the hunting club to hunt on the property as his guests. Answer: B. The court should partition the land into two separate tracts. A court will presume that the devise to the niece and nephew gave them a tenancy in common. Hence, each of them has the right to possess all portions of the property; neither of them has the right to exclusive possession of any party. However, any tenant in common has a right to judicial partition of the property, either in kind or by sale and division of the proceeds. When co-tenants are squabbling and cannot come to any agreement, the remedy of partition terminates the co-tenancy and divides the common property. Since the niece and nephew cannot agree on the use of the land by members of the hunting club, the court will probably partition the property.

*MBE FAVORITE - MPQ* A golf pro entered into an employment contract with a country club to be its golf pro. The agreement specified that the golf pro would run the pro shop and provide private instructions to members from April through September of each year for the next five years, at a monthly salary of $5,000, plus instructional fees. During those months, the club's other instructor was playing on the professional tour and was unavailable. On March 15, the club's manager received an e-mail from the golf pro, stating: "Made the final cut in Sarasota Winter Open. May not be able to get to club by April 1. Could be delayed until 5th if playoff necessary." The club's 'manager asked his attorney whether he should bring an immediate action against the golf pro for breach of contract. Which of the following is the most accurate advice for the manager?

A. Do not file suit; a repudiation must be in a signed writing to be of legal effect. B. Do not file suit; the golf pro has not repudiated the contract. C. File suit, but only if the club has changed its position to its detriment in reliance on the e-mail. D. File suit, because the golf pro has repudiated the contract. Answer: B. The best advice is not to file suit because the golf pro's e-mail does not constitute an anticipatory repudiation. Language may constitute an expression of doubt as to one's ability to perform under the contract without being an outright refusal. This will not be an anticipatory repudiation, but a prospective inability to perform. If there is an anticipatory repudiation, then the non breaching party can (1) sue for damages, (2) contract with a third party, or (3) do nothing. If the fact pattern language amounts to a prospective inability to perform, the innocent party may suspend performance until he receives adequate assurances that performance will be forthcoming. Here, the golf pro's e-mail doe snot constitute an anticipatory repudiation because he merely states that he "may not" get to the club by April 1.

A shopper at a flea market noticed a vase and asked a nearby person how much it cost. The person replied, "One hundred dollars." The shopper paid him the money and took the vase. The person, who was not the owner of the vase but merely a bystander, absconded with the $100. What crime at common law has the bystander committed with respect to the $100?

A. Embezzlement B. Larceny by trick C. Obtaining property by false pretenses D. No crime. Answer: C. The bystander has obtained property by false pretenses. In the crime of false pretenses, the defendant obtains title to the property by means of false representation of a material present or past fact that causes the victim to pass title to his property to the defendant, who knows his representations to be false and intends thereby to defraud the victim. Under the circumstances, by replying that the price of the vase was $100, the bystander implied that he was its owner or was authorized to sell it, which he was not. This false representation induced the victim to pass title to his money to the bystander, who knew the representation to be false. Therefore, he is guilty of obtaining money by false pretenses.

A woman filed a diversity action against a man in federal court in State A, alleging that the man failed to pay a debt due on a valid contract. The man has paid the debt, and has the cancelled checks to prove it. What should the man's attorneys do?

A. File a motion to dismiss for failure to state a claim on which relief can be granted, attaching an affidavit with the cancelled checks. B. File an answer and then a motion for summary judgment, attaching an affidavit with the cancelled checks. C. File only a motion for summary judgment, attaching an affidavit with the cancelled checks. D. File a motion for judgment as a matter of law, attaching an affidavit with the cancelled checks. Answer: B. B is correct. The man has a valid defense, but it is not one of the defenses that can be asserted in a pre-answer Rule 12 motion to dismiss. Therefore, the man accordingly must file an answer. Because the man's defense relies on material outside the pleadings, the man should file a motion for summary judgment, attaching an affidavit with the cancelled checks.

A woman filed a diversity action against a man in federal court in State A, alleging that the man failed to pay a debt due on a valid contract. The man has paid the debt, and has the cancelled checks to prove it. What should the man's attorneys do?

A. File a motion to dismiss for failure to state a claim on which relief can be granted, attaching an affidavit with the cancelled checks. B. File an answer and then a motion for summary judgment, attaching an affidavit with the cancelled checks. C. File only a motion for summary judgment, attaching an affidavit with the cancelled checks. D. File a motion for judgment as a matter of law, attaching an affidavit with the cancelled checks. Answer: B is correct. The man has a valid defense, but it is not one of the defenses that can be asserted in a pre-answer Rule 12 motion to dismiss. Therefore, the man accordingly must file an answer. Because the man's defense relies on material outside the pleadings, the man should file a motion for summary judgment, attaching an affidavit with the cancelled checks.

Several students at a public high school told a teacher that a fellow student was selling illegal drugs to other students at the school. The accused student was called into the principal's officer and informed of the accusations. The student denied everything, but the principal grabbed the student's purse, which was on top of his desk, and opened it. He removed five small transparent plastic bags, each of which contained a white powder, and immediately called the police. The police arrested the student and conducted tests confirming that the white powder was cocaine. The student was charged with possession of a controlled substance with intent to sell. At her trial, the state planned to introduce the bags and their contents into evidence. The student's attorney moved to suppress the evidence. How should the court rule?

A. Grant the motion, because there was adequate time to obtain a warrant. B. Grant the motion, because the principal lacked probable cause. C. Deny the motion, because the principal had a reasonable suspicion that he student was selling drugs. D. Deny the motion, because a principal, acting in loco parentis, has the right to search students whenever he chooses to do so. Answer: C. The motion should be denied. Due to the nature of the school environment, reasonable grounds for a search are a sufficient basis to justify searches by public school officials. Neither a warrant nor probable cause is required. A school search will be held reasonable if: (1) it offers a moderate change of finding evidence of wrongdoing; (2) the measures adopted to carry out the search are reasonably related to the objectives of the search; and (3) the search is not excessively intrusive in light of the age and sex of the student and the nature of the infraction. Here, even though the principal did not have probable cause to believe that the drugs were in the student's purse; the principal did have sufficient reasonable grounds to search her purse.

To punish a gambler who owed him money, a bookie purchased from a demotions expert a small explosive device that he planned to place under the driver's seat of the gambler's automobile. The demolition expert assured the bookie that the explosive would be strong enough to cause severe damage to the legs and ankles of anyone sitting in the front seat of the car, but would under no circumstances be strong enough to kill. The device would go off when the ignition was turned on. On Sunday night, the bookie planted the device in the gambler's 15 year old son came home from school and saw his father's car in the driveway. The son took an extra set of keys from the house to take the car for a joyride, even though the gambler had forbidden his son to drive the car without permission. When the 15 year old turned on the ignition, the explosive device went off and the boy died instantly. If the bookie is charged with the murder of the gambler's son, what is the jury's verdict likely to be?

A. Guilty, because the bookie's act caused the son's death. B. Guilty, because he intended to cause serious bodily harm to the gambler. C. Not guilt, because he did not intend to cause the death of any person. D. Not guilty, because it was not foreseeable that the son would attempt to drive the car. Answer: B. The bookie is guilty because his act, which caused the death of the gambler's son, was committed with an intent to cause serious bodily harm. At CL, murder was the killing of a human being with malice aforethought. In this question, the bookie clearly intended to cause serious bodily harm to the gambler. Under the doctrine of transferred intent, if the actor intended to kill or cause serious bodily harm to one person and caused the death of another person, malice aforethought as to the unintended victim was established. Thus, under the doctrine, the bookie's intent to harm the gambler will be transferred to the gambler's son, and the bookie will be guilty of the murder of the gambler's son.

Owen owned Blackacre in fee simple. He executed a will leaving the property to his sister, Sarah, for life, with remainder to the children of his niece, Norah. At the time of the execution of the will, Norah had one daughter, Donna. The next year, Norah had a son, Sam. A year later, Owen died. Shortly after Owen's death, Donna died in a plane crash, leaving all of her estate to her husband, Harold. One year after Owen's death, Norah had another daughter, Debbi. Shortly thereafter, Sarah, Owen's sister, died. Sixteen months after that, Norah had a second son, Sid. Now who owns Blackacre?

A. Harold, Sam, Debbi, and Sid in equal shares. B. Harold and Sam in equal shares. C. Sam, Debbi, and Sid in equal shares. D. Harold, Sam, and Debbi in equal shares. Answer: D. Harold, Sam, and Debbi take Blackare in equal shares. At Owen's death, Norah had a daughter and a son, making their interests at that time vested remainders in fee simple subject to open to make room for any future children of Norah. When Donna died, her vested remainder passed to Harold. When Debbi was born, she joined Sam and Harold, all three holding vested remainders in fee simple. When Owen's sister Sarah died, her life estate ended, giving the present possessory estate to those three holders of the future interest. Why is Sid left out? Because of the class closing rule: whenever any member of the class is entitled to a distribution, the class closes and the distribution is made then. Any later arrivals to the class lose out entirely. Because Sam, Harold, and Debbi are eligible for a distribution the moment the life tenant (Sarah) dies, they take, and Sid, not yet born, loses out.

The owner of Whiteacre in fee simple leased Whiteacre to a tenant for a term of ten years by properly executed written instrument. The lease was promptly and properly recorded. It contained an option for the tenant to purchase Whiteacre by tendering $250,000 as a purchase price any time "during the term of this lease." One year later, the tenant, by a properly executed written instrument, purported to assign the option to his brother, expressly retaining all of the remaining terms of the lease. The instrument of assignment was promptly and properly recorded. Two years later, the owner contracted to sell Whiteacre to his friend and to convey a marketable title "subject to rights of the tenant under her lease." The friend refused to close because of the outstanding option assigned to the brother. The owner brought an appropriate action against the friend for specific performance. If judgment is rendered in favor of the owner, it will be because the relevant jurisdiction has adopted a rule on a key issue as to which various state courts have split. Which of the following identifies the determinative rule or doctrine upon which the split occurs, and states the position favorable to the owner

A. In a contract to buy, any form of "subject to a lease" clause that fails to mention expressly an existing option means that the seller is agreeing to sell free and clear of any option originally included in the lease. B. Marketable title can be conveyed so long as any outstanding option not mentioned in the purchase contract has not yet been exercised. C. Options to purchase by lessees are subject to the Rule Against Perpetuities. D. Options to purchase contained in a lease cannot be assigned separately from the lease. Answer: D is the correct answer. This is a reading comprehension question and can be answered through the process of elimination. The key issue is that the tenant only has a lease for years, which is not a complete ownership interest, but merely a nonfreehold estate in Whiteacre. The tenant's option to purchase was "during the term of this lease." The tenant does not have a property interest in the option; he only has an alienable interest in the leasehold itself, allowing him to assign his entire interest or to sublet his interest by granting possession of the estate for a term of years that is less than the entire interest. Therefore, A and B can be eliminated immediately because the choices want the reader to erroneously assume a transfer of property has occurred. The tenant had a lease for years that contained an option to purchase, not a contract to buy Whiteacre that contained a lease clause. In addition, marketable title is not at issue because the property was not sold. The issue is whether a leasehold estate can assign its option to purchase while keeping the remaining terms of the lease. C is clearly incorrect because the option to purchase must be exercised within ten years, removing it from any potential rule against perpetuities violation. That leaves D, which correctly states that options to purchase contained in a lease for years cannot be assigned separately from the lease. A, B and C are incorrect.

A landowner contracted in writing to sell a lot to a brother and a sister, as joint tenants, for $60,000. The brother and the sister put up $6,000 as earnest money. Before the closing date, the landowner died. Shortly thereafter, and also before the closing date, a garage on the lot burned down. The garage had a fair market value of $6,500 and was complete loss. After the fire, the brother demanded that the executor of the landowner's estate return the $6,000, because the brother and sister were no longer interested in the property. The executor refused and told the brother that he expected the brother and sister to tender the $54,000 due on the property when the closing date arrived. The brother and sister did not do so. The brother filed suit demanding a refund of the $6,000. The executor countersued, demanding specific performance by the brother and sister or, in the alternative, monetary damages. Absent any applicable statutes, how should the court rule?

A. In favor of the executor, by requiring specific performance of the brother and sister. B. In favor of the executor, by assessing damages against the brother and sister. C. In favor of the brother, by ordering the executor to refund the earnest money. D. The court should rule that the executor is not entitled to either damages or specific performance and that the brother is not entitled to a refund of the earnest money. Answer: A. The court should require specific performance of the brother and sister. When a transfer of land is preceded by a contract for sale, the risk of loss to the property during that time interval is imposed on the buyer in most jurisdictions. Thus, despite a loss due to fire or other casualty (assuming it was not due to the fault of either party), the buyer must still pay the contract price at the closing date unless the contract provides otherwise. Consequently, the brother and sister were still under a duty to tender the amount due on the property. Their failure to do so puts them in breach of contract, allowing the executor to obtain specific performance (payment of the balance due on the contract.)

*MBE FAVORITE - MPQ* A gourmet food company entered into a long-term contract with an airline, under which the food company would supply the airline with a 5 million gourmet dinners over a five-year period at a special rate of $2 per unit. The food company insisted as a term of the contract that the airline agree to purchase from a microwave supplier, and to install in each of its planes, a microwave oven specifically designed to heat frozen dinners, in part because the food company owned considerable stock in the microwave supplier. The contract between the food company an the airline had a clause that authorized "oral modifications by the contracting parties." One month after the contract was signed but before any dinners were delivered, the airline informed the food company that it would have difficulty complying with the provision requiring purchase of the supplier's microwaves because the supplier's products had increased dramatically in price. Subsequent negotiations between the food company and the airline led to an oral agreement to increase the price per dinner to $2.08 per unit and eliminate the supplier's microwave requirement. If the supplier sues the airline for enforcement of the contract, what will be the most likely result?

A. Judgment for the supplier, because under the UCC the oral modification is not valid and the supplier's rights vested when it brought suit. B. Judgment for the supplier, because it was a third-party beneficiary to the original contract and it did not agree to the modification. C. Judgment for the airline, because the contract was between the airline and the food company, and by its terms an oral modification was permissible. D. Judgment for the airline, because the supplier's rights had not vested when the modification took place. Answer: D. The airline will prevail because the supplier's rights had not vested at the time of the modification. The right of the third-party beneficiary do not vest until: (1) it manifests assent in a manner invited or requested by the parties; (2) it learns of the contract and detrimentally relies on it; or (3) it brings a lawsuit to enforce its rights. Until a third party's rights have vested, a modification of the contract can take place without the consent of the third party. The proper analysis is that the supplier's rights have not vested. Even if the supplier was aware of the contract. Moreover, notwithstanding the oral modification clause, while the oral modification will be enforceable between the food company and the airline only if an exception to the SOF applies (e.g. parties admit the modification or perform the contract as modified), it is doubtful that a court would allow the supplier to use the Statute to prevent the parties from admitting their modification. Thus, the modification is valid, and (A) is wrong. (B) is wrong because the supplier's rights had not vested. (C) is a true statement, but (D) is a better answer, because it states the specific reason that the supplier's rights had not vested.

A man asked a coworker who was a wine collector to lend him a bottle of expensive wine to put in his liquor cabinet, because he was inviting his mother over for dinner and wanted to impress her. The coworker permitted the man to take a bottle of wine worth $700 to his apartment for the dinner as long as he returned it the next morning. As he had planned all along, the man instead invited his girlfriend over for a romantic dinner, at which they drank the bottle of wine. The next day the man told his coworker that he had been mugged on his way home and that the muggers made off with the wine. Suspicious, the coworker found the empty wine bottle in the man's trash fro his apartment. If the man is charged with theft in a common law jurisdiction, of which theft offense is he most likely to be convicted?

A. Larceny by trick B. Larceny C. Embezzlement D. No crime. Answer: A. The man can be convicted of larceny by trick. The owner of the wine gave possession of the bottle of wine to the man, but clearly did not give up ownership, because possession was transferred on the coworker's explicitly promise to return it the next day. Therefore, a theft crime of some sort was committed, but that crime is not pure common law larceny because there was no trespassory taking. If the man obtained possession of the wine by fraud or misrepresentation, then he is guilty of larceny by trick. Because it apparently was the man's intent to drink the wine all along, the possession was obtained by fraud and the crime is larceny by trick, not embezzlement (the crime which would have occurred if he obtained rightful possession).

During a heated argument, a man punched his female coworker in the stomach after learning that she had been awarded "Employee of the Month." Angered, the woman responded by stabbing the man with a letter opener, which severed his aorta and caused his death. The police arrested the woman. After receiving her Miranda rights, she confessed to killing her co-worker, but stated that she had not previously held any grudge against him. When asked why she stabbed him, she stated, "He just made me so made when he said that I was a terrible employee and that he deserved the reward instead of me, and I just lost it when he punched me." The district attorney charged the woman with homicide. If the jury believes the woman's statement, of what crime is she most likely to be found guilty?

A. Murder B. Voluntary manslaughter C. Involuntary manslaughter D. No homicide crime Answer: B. The woman would most likely be found guilty of voluntary manslaughter in light of the provoking event. Adequate provocation will reduce a killing to voluntary manslaughter if the D was both reasonably provoked and actually provoked. The woman would also have to offer evidence that the insults and taunting, followed by a sudden punch in the stomach, would cause an ordinary person to be provoked, which is likely to be the case. Hence, her statement in conjunction with the facts makes this the best answer.

A police offer partrolling at 2 am heard a gunshot and saw a figure leap over a hedge in front of a condominium. Although it was dark, the officer could see that he was carrying a briefcase in one hand a gun in the other. Just then, someone limped out of the building and pointed at the figure, yelling, "He shot me!" The officer then twice commanded the figure to stop and fired three warning shots in the air. The person kept running, so the officer fired a fourth shot, bringing the person down. The perosn was later identified as a robber. If the officer is charged with battery, what is the likley veridct of the jury?

A. No guilty, because he had a right to use deadly force as necessary to stop a fleeing felon posing a threat of serious bodily harm to others. B. Not guilty, because a police officer cannot commit a crime while acting within the scope of his duty. C. Guilty, because a police officer does not have the right to use deadly force to prevent a getaway. D. Guilty, because he did not see the robber steal anything. Answer: A. The officer should be found not guilty of battery. Police officers are generally entitled to use whatever force is reasonably required, including deadly force, to apprehend or prevent the escape of a felon who poses a threat of serious bodily harm to the officer or others. Here, the robber had apparently just shot someone and was escaping with his gun, justifying the use of deadly force.

A homeowner decided to destroy his home by fire in order to collect the insurance. A neighbor's house was located a short distance from the homeowner's home. The homeowner knew that there was a strong wind blowing towards the neighbor's home; while he did not want to burn the neighbor's home, he nevertheless set fire to his own home. The fire department was unable to save the homeowner's house. They did manage to put out the fire moments before it spread to the neighbor's home, which suffered damage form smoke and soot. The jurisdiction's arson statute covers burning one's own dwelling as well as the dwelling of another, but is otherwise unchanged from the common law. If the homeowner is charged with attempted arson of the neighbor's home, is he likely to be found guilty?

A. No, because he did not intend to burn the neighbor's house. B. No, because the fire was put out before any part of the neighbor's home was burned. C. Yes, because he intended to burn his own home and took a substantial step toward burning the neighbor's house. D. Yes, because he acted with malice and took a substantial step toward burning the neighbor's house. Answer: A. The homeowner will not be found guilty because he did not have the requisite mental state. To convict a person for an attempted crime, the prosecution must establish that the defendant had an actual specific intent to cause the harm prohibited by the statute and committed an act beyond mere preparation in furtherance of that intent. Those elements - specific intent and act - are required regardless of the mental state required by the target offense. A person who took a substantial step towards commission of the crime but was only reckless with respect to the target offense could not be found guilty of attempt. The homeowner did not intend to burn the neighbor's home. Therefore, he cannot be guilty of attempted arson of the neighbor's home.

A state university adopted a new regulation prohibiting certain kinds of speech on campus. Students, staff, and faculty convicted by campus tribunals of violating the regulation were subject to penalties that included fines, suspensions, expulsions, and termination of employment. The regulation was widely unpopular, and there was a great deal of public anger directed toward the two tenured professors who had drafted and promoted it. The following year, the state legislature approved a severable provision in the appropriations bill for the university declaring that none of the university's funding could be used to pay the two professors, who were specifically named in the provision. In the past, the professors' salaries had always been paid from funds appropriated to the university by the legislature, and the university had no other funds that could be used to pay them. If the professors challenge the constitutionality of the appropriations provision, is the court likely to uphold the provision?

A. No, because it amounts to the imposition by the legislature of a punishment without trial. B. No, because it was based on conduct the professors engaged in before it was enacted. C. Yes, because the Eleventh Amendment gives the state legislature plenary power to appropriate state funds in the manner that it deems most conducive to the welfare of its people. D. Yes, because the full faith and credit clause requires the court to enforce the provision strictly according to its terms. Answer: A. The provision is a bill of attainder in violation of Article I, Section 10, Clause 1 of the Constitution. A bill of attainder is a law that provides for the punishment of a particular person without trial. The challenged provision satisfies this definition because it deprives two named professors of their salaries and thus their employment. Wrong: Answer B is incorrect. It is true that the court is likely to strike down the provision, but this answer misstates the basis for this conclusion. The fact that the professors' conduct preexisted the state law would be significant if the state law provided for a criminal penalty; it would then be unconstitutional as an ex post facto law in violation of Article I, Section 10, Clause 1 of the Constitution. The ex post facto clause, however, does not apply to laws attaching civil consequences to past conduct. In this case, the provision does not alter the criminal law but provides for the punishment of particular people, without trial, by depriving the two named professors of their salaries. As such, it is an unconstitutional bill of attainder in violation of Article I, Section 10, Clause 1 of the Constitution, regardless of the timing of the professors' conduct. Answer C is incorrect. The Eleventh Amendment provides for state sovereign immunity from certain kinds of adjudications. It does not extend legislative authority of any kind to the states. The provision in this case deprives two named professors of their salaries without affording them a trial. As such, the court will strike it down as a bill of attainder in violation of Article I, Section 10, Clause 1 of the Constitution. Answer D is incorrect. The full faith and credit clause (Article IV, Section 1 of the Constitution) does not insulate state laws from constitutional challenge. It merely requires state courts to accord due authority to the laws of other states. The provision in this case deprives two named professors of their salaries without affording them a trial. As such, the court will strike it down as a bill of attainder in violation of Article I, Section 10, Clause 1 of the Constitution.

A microbrewery and a farming operation entered into a written contract for the sale of barley. The microbrewery agreed to buy from the farm "all the barley that the microbrewery requires" in its manufacture of beer, for a period of five years, at a mutually agreed-upon price. Under the contract, the microbrewery would place its orders on the first of each month and the barley would be delivered within five business days. For the first two years of the contract, the microbrewery placed its orders on the first of each month for either four or five barrels of barley. At the beginning of the third year of the contract, an article about the microbrewery appeared in a national newspaper, causing its popularity to soar. The following month, the microbrewery placed an order for 20 barrels of barley. The farm could not meet the increased demand and refused to deliver the 20 barrels. The microbrewery sued the farm for breach of contract. Will the microbrewery be successful in its suit?

A. No, because its order for 20 barrels of barley was unreasonably disproportionate to its previous orders over a two-year period. B. No, because the quantity term in a contract for a sale of goods must be certain. C. Yes, because its order for 20 barrels of barley reflected its actual requirement for that month. D. Yes, because its change in demand was unforeseeable when the parties entered into the contract. Answer: A. The farm will prevail. Under UCC section 2-306(1), quantities subject to requirements contracts may not be unreasonably disproportionate to any stated estimate or, in the absence of any stated estimate, to any normal or otherwise comparable prior requirements.

A state adopted a rule denying admission to its bar to anyone who was currently or had previously been a member of a subversive group. The state's bar application form was modified to ask applicants whether they were or hard previously been members of any subversive organization. An applicant refused to answer the question and was denied bar admission on that basis. The applicant challenged the decision, arguing that the question infringed upon his freedom of association. Is the applicant likely to prevail?

A. No, because membership in a subversive group constitutes endorsement of the group's illegal activities. B. No, because the Constitution does not apply to the bar. C. Yes, because denying bar admission based on any association with a subversive organization violates the First Amendment. D. Yes, because denying bar admission based solely on past membership in a subversive organization violates the First Amendment. Answer: D. The U.S. Supreme Court has held that the freedom of association protected by the First Amendment prohibits a state from inquiring about an individual's associations in order to withhold a right or benefit because of the individual's beliefs. Although the state has a legitimate interest in determining the character and profession competence of bar applicants, the Court has held that the state has other means of making these determinations that are less restrictive of First Amendment freedoms.

In recent years, several large corporations incorporated and headquartered in a particular state have suddenly been acquired by out-of-state corporations that have moved all of their operations out of this state. Other corporations incorporated and headquartered in this particular state have successfully resisted such attempts at acquisition. In an effort to preserve jobs in the state and to protect its domestic corporations against their sudden acquisition by out-of-state purchasers, the state legislature enacts a statute governing acquisitions of shares in all corporations incorporated in the state. This statute requires that any acquisition of more than 25% of the voting shares of a corporation incorporated in the state that occurs over a period of less than one year must be approved by the holders of record of a majority of the shares of the corporation as of the day before the commencement of the acquisition of those shares. The statute expressly applies to acquisitions of the state's corporations by both in-state and out-of-state entities. Assume that no federal statute applies. Is this state statute constitutional?

A. No, because one of the purposes of the statute is to prevent out-of-state entities from acquiring corporations incorporated and headquartered in the state. B. No, because the effect of the statute will necessarily be to hinder the acquisition of the state's corporations by other corporations, many of whose shareholders are not residents of the state and, therefore, it will adversely affect the interstate sale of securities. C. Yes, because the statute imposes the same burden on both in-state and out-of-state entities wishing to acquire one of the state's corporations, it regulates only the acquisition of the state's corporations, and it does not create an impermissible risk of inconsistent regulation on this subject by different states. D. Yes, because corporations exist only by virtue of state law and, therefore, the negative implications of the commerce clause do not apply to state regulations governing their creation and acquisition. Answer: C is the correct answer. The statute does not classify corporations that can acquire one of the state's corporations, but only regulates the acquisition of those corporations. Therefore, it does not violate the privileges and immunities clause or discriminate against interstate commerce. Additionally, because corporations are created by state law, this statute does not create an impermissible risk of inconsistent regulation on this subject by different states. No other state can regulate how the state's corporations can be acquired.

A mother purchased an expensive television from an appliance store for her adult son. Two years after the purchase, a fire started in the son's living room in the middle of the night. The fire department concluded that the fire had started in the television. No other facts are known. The son sued the appliance store for negligence. The store has moved for summary judgment. Should the court grant the store's motion?

A. No, because televisions do not catch fire in the absence of negligence. B. No, because the store sold the television. C. Yes, because the son is not in privity with the store. D. Yes, because there is no evidence of negligence on the part of the store. Answer: D. The son is suing in negligence, not in strict liability. To make out a prima facie case in negligence, the son must introduce evidence that the store was negligent. However, the son has not pointed to any negligent action or omission by the store. This is not an appropriate case for res ipsa loquitur, because the manufacturer, rather than the store, may have been negligent or the negligence may have occurred after the sale (for example, during a repair or while the television was being used by the son).

Toxic materials being transported by truck from a manufacturer's plant to a warehouse leaked from the truck onto the street a few miles from the plant. A driver lost control of his car when he hit the puddle of spilled toxic materials on the street, and he was injured when his car hit a stop sign. In an action for damages by the driver against the manufacturer based on strict liability, is the driver likely to prevail?

A. No, because the driver's loss of control was an intervening cause. B. No, because the driver's injury did not result from the toxicity of the materials. C. Yes, because the manufacturer is strictly liable for leaks of its toxic materials. D. Yes, because the leak occurred near the manufacturer's plant. Answer: B. Strict liability in this situation would be based on the abnormally dangerous nature of the toxic materials. But a successful strict liability action requires that the risk that materializes be the same risk that led courts to label the activity "abnormally dangerous" in the first place. Here, the toxicity of the materials did not contribute to the driver's injury, so his only cause of action would be in negligence.

A city ordinance prohibited individuals from picketing in residential neighborhoods unless the picketing related to the neighborhood zoning requirements. This exception to the ordinance was adopted in response to local citizens' strong views about proposed rezoning of residential neighborhoods. A group that wished to picket in front of a business owner's home because of the business owner's employment practices challenged the ordinance as unconstitutional under the First Amendment. Will the group's challenge likely prevail?

A. No, because the ordinance is a content neutral regulation of speech. B. No, because the ordinance regulates conduct rather than speech. C. Yes, because the ordinance irrationally discriminates between different types of protesters. D. Yes, because the ordinance is a content-based regulation of speech. Answer: D. The ordinance is a content-based regulation of speech because it permits an expressive activity (picketing) on one subject (neighborhood zoning requirements) and prohibits it on all other subjects. Such a content-based restriction on expression presumptively violates the freedom of speech protected by the First Amendment. To justify a content-based restriction, the government must satisfy strict scrutiny, proving that the restriction is necessary to serve a compelling government interest. The city would be unable to meet that burden in this case.

The elderly owner of a small furnished vacation cottage who had not used it for several years decided to rent it out to vacationers. His first tenants, a newlywed couple, agreed to pay $400 a month stay, although the owner apologized that he had not been able to clean the cottage before renting it because he had not been there for several years. When they arrived at the cottage, the wife started carrying the luggage up the wooden steps of the house and one of the steps collapsed, seriously injuring her. On inspection it was revealed that the steps had rotted. Although no one had been aware of the defect, a reasonable inspection would have disclosed it. Is the owner liable to the wife for her injuries?

A. No, because the owner neither knew nor had reason to know that the steps were rotted. B. No, because the owner told the tenants that he had not been to the house for several years and so was not aware of its current condition, and the wife could have discovered the defect if she had made a reasonable inspection. C. Yes, because the landlord warrants that premises are free of latent defects. D. Yes, because the lease is for one month and the house is furnished. Answer: D. The owner is liable to the wife. Landlords are liable for latent defects even if they neither knew nor should have known of the defect if the lease is for a short term and the property is furnished. This is an exception to the general rule that a landlord is not liable for latent defects unless the landlord either knew or had reason to know of the defects.

A man beat his live-in girlfriend and fled. The girlfriend called the police and told them about the beating. She also told them that the man likely fled to his best friend's house. The police quickly obtained a valid arrest warrant for the man and went to the friend's house. The police quickly obtained a valid arrest warrant for the man and went to the friend's house a few hours after the beating. On arriving, the police notice that a car registered to the man was parked nearby. They knocked and the friend answered the door. The friend told the police that the man was not there. The police pushed past the friend and began searching for the man. They found the man hiding in a closet and arrested him. On searching the man after his arrest, police found cocaine in a small metal box in the man's pants pocket. The man was charged with assault and possession of cocaine. In a pretrial motion, the man moved to suppress the cocaine, claiming that it was the fruit of an unconstitutional arrest. Should the court grant the motion?

A. No, because the police found the cocaine after executing a valid arrest warrant. B. No, because the search of the friend's house was valid under the community caretaker exception. C. Yes, because the police entered the house without a search warrant or consent. D. Yes, because the small metal box could not have felt like a weapon or contraband. Answer: A. The court should not grant the motion. The police may search a person after making a valid arrest. The arrest here was valid because the police had a valid arrest warrant. While the police should have obtained a search warrant to search for the man in the friend's home, the man does not have standing to complain of the illegal search. One may not raise a violation of another's constitutional rights at a Fourth Amendment suppression hearing. A person generally does not have standing to complain about a warrantless search of another's home unless the home was also his home or he was at least an overnight guest in the home. Here, the facts say that the man lived with his girlfriend, fled to the friend's house, and had been there a few hours. Thus, he lacks standing to complain about the warrantless search.

A fugitive was wanted for murder. The authorities offered the following reward: "$20,000 to anyone who provides information leading to the arrest and conviction of this fugitive." A private detective knew of the reward, located the fugitive, and brought him to the authorities, who arrested him. The authorities then determined that while the fugitive had, in fact, committed the crime, he had been directed to commit the crime by his boss. The authorities and the fugitive then agreed that in exchange for the fugitive's testimony against his boss, all charges against the fugitive would be dropped. The fugitive testified and was released. The authorities refused to pay the reward to the private detective on the ground that the fugitive was never convicted. Would the private detective be likely to prevail in a breach of contract action against the authorities?

A. No, because the private detective failed to notify the authorities that he had accepted the reward offer. B. No, because the express condition set out in the reward were not met. C. Yes, because the authorities' agreement with the fugitive was against public policy. D. Yes, because the authorities themselves prevented the conviction of the fugitive. Answer: D. A performance that is subject to an express condition cannot become due unless the condition occurs or its nonoccurrence is excused. The second, the conviction, did not occur. Its nonoccurrence is excused, however, under the doctrine of prevention, which requires that a party refrain from conduct that prevents or hinders the occurrence of a condition.

A sailor filed suit in federal court against several boat manufacturers, who all used asbestos products on the ships they made, alleging that she had been exposed to asbestos while working in the naval yard. Discovery revealed that ships that contained asbestos manufactured by each of the named defendants had, at various times, been at the naval yard where the sailor had worked over the course of her employment. One of the named boating manufacturers moved for summary judgment, arguing that the sailor had no evidence that she had been exposed to asbestos products made by this particular boating manufacturer. In its motion for summary judgment, the boating manufacturer did not provide any evidence contradicting its presence in the naval yard nor did it provide any support to negate the evidence put forth by the sailor. Should the court grant the boating manufacturer's motion for summary judgment?

A. No; the court should deny the motion for summary judgment because the boating manufacturer, as the moving party, failed to make the required affirmative evidentiary showing in support of its motion. B. No; the court should deny the motion for summary judgment because the boating manufacturer not only failed to show that no evidence in the record supported the sailor's case, but also failed to make an affirmative evidentiary showing in support of its motion. C. Yes; the court should grant the motion for summary judgment because it is clear that there is no genuine dispute of material facts between the parties. D. Yes; the court should grant the motion for summary judgment because the sailor acted in bad faith when she filed the suit without knowledge of which boating manufacturers were present in the naval yard when she was working. Answer: B is the correct answer. A defendant who moves for summary judgment may win the motion either by demonstrating that no evidence in the record supports the plaintiff's case, or by a showing of affirmative evidentiary support. In this case, there was evidence in the record that ships containing asbestos manufactured by the moving manufacturer had been at the same naval yard where the sailor had worked during the time she was present. While that evidence alone does not establish the sailor's exposure to the moving manufacturer's asbestos, it is evidence that supports the sailor's case. Therefore the moving manufacturer, in its motion for summary judgment, would have had to make an affirmative showing to undermine the evidence present in the record.

A patient brought a negligence action in federal district court in State A against a dentist and a nurse following a root canal procedure the dentist and nurse performed. The patient sought more than $75,000 in compensatory damages from both the dentist and the nurse. The patient is a citizen of State A; the dentist is a citizen of State B; and the nurse is a citizen of State A. The nurse then moved to dismiss the case for lack of complete diversity between the parties. In opposition, the patient argued that the court can assert supplemental jurisdiction over her claim against the nurse. Should the district court grant the nurse's motion to dismiss?

A. No; the district court should deny the motion to dismiss because the patient's claim against the dentist is a civil action within the original jursidiction of the federal courts and the courts can assert supplemental jurisdiction over the claim against the nurse because it is part of the same case or controversy as the patient's claim against the dentist. B. Yes; the district court should grant the nurse's motion to dismiss because even though the patient's claim against the dentist is a civil action within the original jurisdiction of the federal courts, the court does not have supplemental jurisdiction over the claim against the nurse because it doe snot arise from the same case or controversy as the claim against the dentist. C. Yes; the district court should grant the nurse's motion to dismiss because there is no diversity between the parties and therefore the court lacks diversity jurisdiction. D. No; the district court should deny the motion to dismiss because the patient's claim against the dentist is a civil action within the original jurisdiction of the federal courts and the judicial economy would be served by litigating the patient's claims against the dentist and the nurse together. Answer: C. The nurse's presence in the claim prohibits the court from having diversity jurisdiction over the patient's claims against the dentist and the nurse. A plaintiff, generally, cannot use supplemental jurisdiction to overcome a lack of diversity in a diversity case.

A landscaper entered into a written contract with a developer to landscape a 30-house subdivision at a price of $4,000 for each house. The contract provided for payment of the $120,000 only on completion of the landscaping for all the houses. After completing 20 houses, the landscaper demanded payment of $80,000. The developer refused. The landscaper then walked off the job without doing any landscaping on the other 10 houses. The developer refuses to pay the landscaper. If the landscaper sues the developer, what damages should the court award the landscaper?

A. Nothing, because payment was expressly conditioned on completing the landscaping of all the houses. B. Nothing, because the landscaper's breach is material. C. $80,000, less the developer's damages resulting from the breach. D. The amount of the landscaper's anticipated profit on the 20 houses completed. Answer: C. The landscaper may recover $80,000 less the developer's damages resulting from the breach. A contract is divisible if it is possible to apportion the parties' performances into corresponding pairs.

A politician, from State B, filed a lawsuit against a stockbroker, from State A, in state court in State A, seeking over $75,000 as compensation for damages incurred in a car accident. The jury found, pursuant to a special verdict, that the stockbroker had not been negligent, and awarded no damages to the politician. Six months later, an assistant, who also had been involved in the car accident, filed a jurisdictionally valid lawsuit against the stockbroker in federal court in State A. Which of the following is true?

A. The assistant will be precluded from arguing that the stockbroker was negligent. B. The assistant will not be precluded from arguing that the stockbroker was negligent. C. The assistant's lawsuit will be dismissed on grounds of issue preclusion. D. The assistant's lawsuit will be dismissed on grounds of claim preclusion. Answer: B is correct. Here, the assistant will not be precluded from arguing that the stockbroker was negligent because the assistant was not a party in the prior law suit; non-mutuality collateral estoppel cannot be asserted against someone who was not a party in the first lawsuit - doing so would violate due process.

A husband conveyed his home to his wife for life, remainder to his daughter. There was a $20,000 mortgage on the home, requiring monthly payment covering interest to date plus a portion of the principal. Which of the following statements about the monthly payment is correct?

A. The wife must pay the full monthly payment. B. The wife must pay a portion of the monthly payment based on an apportionment of the value between her life estate and the daughter's remainder. C. The wife must pay the portion of the monthly payment which represents interest. D. The daughter must pay the full monthly payment. Answer: C. The husband's conveyance created a life estate in the wife and a vested remainder in the daughter. As a life tenant, the wife has a duty not to commit voluntary or permissive waste relevant to the property during her life tenancy. A failure to keep the property in repair, pay taxes on the property, or pay interest on any mortgage on the property all constitute permissive waste. Although the wife's duty not to commit permissive waste requires her to pay the interest on the mortgage, she has no obligation to pay the principal.

On March 2, a landowner and a builder orally agreed that the builder would erect a boathouse on the landowner's lot and dig a channel from the boathouse, across a neighbor's lot, to a lake. The neighbor had already orally agreed with the landowner to permit the digging of the channel across the neighbor's lot. The builder agreed to begin work on the boathouse on March 15, and to complete all the work before June 1. The total price of $10,000 was to be paid by the landowner in three installments: $2,500 on March 15; $2,500 when the boathouse was completed; $5,000 when the builder finished the digging of the channel. Assume that the landowner paid the $2,500 on March 15 and that the builder completed the boathouse according to specifications, but that the landowner then refused to pay the second installment and repudiated the contract. Assume further that the absence of a writing is not raised as a defense. Which of the following is correct?

A. The builder has a cause of action against the landowner and his damages will be $2,500. B. The builder can refuse to dig the channel and not be liable for breach of contract. C. The builder can refuse to dig the channel, not be liable for breach of contract, and have a cause of action against the landowner for which his damages will be $2,500. D. The builder can refuse to dig the channel but he will liable for breach of contract. Answer: B. Under the doctrine of anticipatory repudiation, an unequivocal statement of unwillingness or inability to perform a future contractual obligation, if material, may be treated as a total breach of that obligation. Once a repudiation occurs, the non-repudiating party is discharged from any further performance under the contract and may immediately recover damages for total breach. When a contractor is injured due to an owner's total breach of a construction contract, the contractor generally can recover his expected profit on the entire contract along with any labor and material expenses incurred up until the time that he learned of the owner's breach, minus any progress payments made by the owner. Although the landowner repudiated the agreement just after the second $2,500 progress payment became due, the builder would be entitled to obtain as damages not only the progress payment but also his expected profit on the entire contract. Because these damages would exceed $2,500, answers A and C are incorrect. The builder is also entitled to withhold his remaining performance under the contract and will not be considered in breach. Therefore, answer D is incorrect, making B the correct answer.

On March 1, a seller orally agreed to sell his land, Homestead, to a buyer for $46,000 to be paid on March 31. The buyer orally agreed to pay $25,000 of the purchase price to a creditor of the seller in satisfaction of a debt which the seller said he had promised to pay the creditor. On March 10, the buyer dictated the agreement to his secretary but omitted all reference to the payment of the $25,000 to the creditor. In typing the agreement, the secretary mistakenly typed in $45,000 rather than $46,000 as the purchase price. Neither the buyer nor the seller carefully read the writing before signing it on March 15. Neither noticed the error in price and neither raised any question concerning omission of the payment to the creditor. In an action by the creditor against the buyer for $25,000, which of the following is correct?

A. The buyer could successfully raise the Statute of Frauds as a defense because the agreement was to answer for the debt of another. B. The buyer could successfully raise the Statute of Frauds as a defense because the agreement was for the sale of an interest in land. C. The buyer could successfully raise the Statute of Frauds as a defense because the agreement was to answer for the debt of another and the agreement was for the sale of an interest in land. D. The buyer cannot raise a Statute of Frauds defense. Answer: Answer D is correct. Under the Statute of Frauds, promises made to answer for the debt of another generally fall within the suretyship clause of the statute and therefore must meet the writing and signing requirements of the statute. However, the suretyship clause does not apply where the surety promise was made to the principal (seller) and not to the obligee (the creditor). Contracts for the sale of an interest in land fall within the statute of frauds as well. The promise that the creditor is seeking to enforce, however, is not part of a contract for the sale of an interest in land but rather a promise to repay a $25,000 debt owed to the creditor by the seller. Therefore the buyer cannot raise a Statute of Frauds defense.

An owner conveyed her property to a buyer, who put the deed in his suitcase and took off for a five-month tour of the world. The owner, knowing that the buyer had left the country, sold the property to an investor who was not aware of the previous transaction. The investor did not record her deed. When the buyer returned from his trip, he recorded his deed. A month later, the investor conveyed the property to a developer. The developer knew that the buyer held a deed to the property but completed the transaction anyway. Instead of recording, however, the developer immediately filed an appropriate action against the buyer and against the investor to determine ownership of the property. The property is situated in a jurisdiction containing the following statute: "Any conveyance of an interest in land, other than a lease for less than one year, shall not be valid against any subsequent purchaser for value, without notice, unless the conveyance is recorded." How should the court rule?

A. The buyer has rights superior to those of both the investor and the developer. B. The investor has rights superior to those of both the buyer and the developer. C. The developer has rights superior to those of both the buyer and the investor. D. The developer has rights superior to those of the investor, but the buyer has rights superior to those of the developer. Answer: C. The court will rule that the developer has rights superior to those of the other parties. Under a notice statute, which is the statute applicable here, the general rule is that a subsequent bona fide purchaser ("BFP") will prevail over a prior grantee who failed to record before the BFP's purchase; if the prior grantee has previously recorded, the subsequent purchaser ordinarily will be deemed to have record notice of the prior conveyance and will not be a BFP. Under the shelter rule, a person who takes from a BFP will prevail against any interest that the transferor-BFP would have prevailed against.

A man and a woman who owned two adjacent properties executed and recorded reciprocal easements to five-foot strips of land on the border of their properties for the purpose of constructing a 10-foot-wide common driveway to their street, on which each of them had only five feet of frontage. After the driveway had been constructed, the town constructed a new road abutting the woman's property in the rear. The woman constructed a new driveway to that new street, and soon thereafter ceased using the common driveway. One year later, the woman sold the property to a buyer, who used the new driveway for four years and did not use the old one. The man then commenced using the entire 10 feet of the old driveway by parking his cars on both sides, and the buyer commenced an action to prevent the man's use of the five feet of the driveway on the buyer's land. The period for acquiring title by adverse possession in the jurisdiction is five years. What is the likely outcome of the action?

A. The buyer will prevail because nonuse is insufficient to cause abandonment of the easement. B. The buyer will prevail because he is a bona fide purchaser. C. The man will prevail because the easement was abandoned by the woman and the buyer through their use of only the new driveway. D. the man will prevail because the period required for adverse possession is five years. Answer: A. The Buyer will prevail. The easement in this case is an easement by grant. That easement was not used for five years, but there were neither acts which would have terminated that easement by adverse possession, nor was there a manifested intent to abandon. Exam Tip: Don't be fooled by very longs periods of nonuse in the facts - make sure there is an act showing intent; otherwise, there can be no termination.

Thirty years ago, the original owner of Greenacre, a lot contiguous to Blueacre, in fee simple, executed and delivered to his neighbor an instrument in writing which was denominated "Deed of Conveyance." In pertinent part it read, "[The owner] does grant to [the neighbor] and her heirs and assigns a right-of-way for egress and ingress to Blueacre." If the quoted provision was sufficient to create an interest in land, the instrument met all other requirements for a valid grant. The neighbor held record title in fee simple to Blueacre, which adjoined Greenacre. Twelve years ago the owner's son succeeded to the original owner's title in fee simple in Greenacre and seven years ago the neighbor's daughter succeeded to the neighbor's title in fee simple to Blueacre by a deed which made no mention of a right-of-way or driveway. At the time the neighbor's daughter took title, there existed a driveway across Greenacre which showed evidence that it had been used regularly to travel between the main road and Blueacre. Blueacre did have frontage on a side road, but this means of access was seldom used because it was not as convenient to the dwelling situated on Blueacre as was the main road. The driveway originally was established by the neighbor. The neighbor's daughter has regularly used the driveway since acquiring title. The period of time required to acquire rights by prescription in the jurisdiction is ten years. Six months ago the son notified the neighbor's daughter that the son planned to develop a portion of Greenacre as a residential subdivision and that the daughter should cease any use of the driveway. After some negotiations, the son offered to permit the daughter to construct another driveway to connect with the streets of the proposed subdivision. The daughter declined this offer on the ground that travel from Blueacre to the main road would be more circuitous. The neighbor's daughter brought an appropriate action against the son to obtain a definitive adjudication of the respective rights of the daughter and the son. In such lawsuit the son relied upon the defense that the location of the easement created by the grant from the original owner to the neighbor was governed by reasonableness and that the son's proposed solution was reasonable. The son's defense should?

A. fail, because the location had been established by the acts of the neighbor and the original owner. B. fail, because the location of the easement had been fixed by prescription. C. prevail, because the reasonableness of the son's proposal was established by the neighbor's daughter's refusal to suggest any alternative location. D. prevail, because the servient owner is entitled to select the location of a right-of-way if the grant fails to identify its location. Answer: A is correct. The easement described in the facts is an easement appurtenant - one that benefits a specific piece of land. The creation of an easement appurtenant requires two pieces of property: a dominant estate and a servient one. This easement will automatically run with the land, and after being recorded for the first time does not need to be re-identified in any deeds accompanying later conveyances. The owner of the servient estate (the estate that the easement crosses) has the right to select the location of the easement. The only limit on such selection is that the location chosen must be a reasonable. In this case, the parties do not dispute whether the original owner of the servient estate reasonably chose its location. Instead, the son attempts to argue that moving the easement to a new location is permissible as long as the new location is selected reasonably. The easement, however, has already been reasonably located, and the servient estate does not possess any inherent right to relocate it. Therefore, answer A is correct.

A buyer contracted to purchase from a seller a corner lot with a house. The contract gave the street address of the property as 235 East Madison Street and additionally gave teh following metes and bounds: "East 80 feet along Madison Street, then south 80 feet along Jensen Street, then east 80 feet, then north 80 feet." A casual inspection of the lot and a careful reading of the metes and bounds description would make it clear that the third call should have read "west 80 feet" instead of "east 80 feet." Prior to the closing date, the seller died, and when the buyer appeared on the closing day with a certified check in the amount of the purchase price, the seller's personal representative refused to convey the parcel or to accept the money from the buyer. The representative claimed that the buyer had no rights in the parcel because the contract description was so defective as to the call into question what was to be conveyed. If the buyer sues the seller's estate for specific performance, what is the likely result?

A. The buyer wins, because the fourth call prevails over the third call in a legal description. B. The buyer wins, because equity will not allow a forfeiture based on a technicality. C. The buyer loses, because metes and bounds is an improper way to legally describe land. D. The buyer loses, because the street names do not provide an adequate legal description of land. Answer: B. Buyer will win because equity does not like forfeitures based on technicalities. For the description of land in a land sale contract or deed to be adequate, it must accurately identify the property. As long as it is clear what property is the subject of the contract, the description is acceptable, even tough it may contain errors. A court of equity may grant specific performance. The court will look at the description, together with the address of the land noted in the contract. Theses, taken together, make it clear that call three should have read "west" rather than "east," and that error will not prevent specific performance. MAJORITY - allow a court of equity to consider all available evidence in determining whether to grant specific performance. MINORITY - Do not allow street addresses to be considered in determining wether a description of land is adequate.

On September 1, an art collector offered to sell one of his expensive paintings to a buyer. The buyer, who was a friend of the collector, wanted a few days to make up her mind, so the collector and the buyer decided that the collector would keep his offer to her open until September 8 in exchange for a payment of $5. Later that week an art investor tendered to the collector double what he was asking for the painting. On the morning of September 8, the buyer telephoned the collector to tell him that she wanted the painting but his phone was out of order, so she wrote out a check for the agreed-on amount and dropped it into a mailbox before leaving town. On September 9 the collector, not having heard from the buyer, sold the painting to the investor. Who is entitled to the painting?

A. The buyer, because she had a valid option contract with the collector and effectively accepted the collector's offer to sell the painting. B. The investor, because $5 is inadequate consideration for an option contract involving an expensive purchase such as this one. C. The investor, because the collector sold the painting to the investor on September 9 before receiving the buyers check. D. The collector, because a sale of goods priced at $500 or more must be in writing to be enforceable. Answer: C. The investor owns the painting. While it is true that the buyer had a valid option contract with the collector, she did not effectively accept the collector's offer to sell the painting when she dropped the check into the mailbox on September 8. The MAJORITY VIEW is that acceptance of an option is effective only when received by the offeror, so the usual "mailbox rule" does not apply to make the acceptance effective on dispatch. This means that the buyer did not effectively accept the option within the stated period, i.e., by September 8. When, as here, the time the offer will remain open is specified in the option, if it is not accepted within that time, the offer terminates due to lapse of time.

On January 15, a carpenter agreed to repair a homeowner's house according to certain specifications and to have the work completed by April 1. On March 1, the homeowner's property was inundated by flood waters which did not abate until March 15. The homeowner could not get the house in a condition which would permit the carpenter to begin the repairs until March 31. On that date the carpenter notified the homeowner that he would not repair the house. Which one of the following facts, if it was the only one true and known to both parties on January 15, would best serve the carpenter as the basis for a defense in an action brought against him by the homeowner for breach of contract?

A. The carpenter's busy schedule permitted him to work on the homeowner's house only during the month of March. B. Any delay in making the repairs would not seriously affect the homeowner's use of the property. C. The cost of making repairs was increasing at the rate of 3 percent a month. D. The area around the homeowner's property was frequently flooded during the month of March. Answer: Answer A is correct. A party's performance under a contract may be excused on grounds of supervening impracticability where the performance is made impracticable without his fault due to the occurrence of an event, the non-occurrence of which was a basic assumption on which the contract was made. If the delay resulting from the impracticability is not material, it may operate to suspend but not discharge the parties' remaining obligations under the contract. Since the flood was an act of God that related to a basic assumption of the agreement to repair the homeowner's house, it operated to excuse any resulting delay in completing the requested work. Whether or not the delay was sufficiently severe to discharge the parties' remaining obligations under the contract depends on whether circumstances have so changed that performance will become materially more burdensome than the agreed-upon performance. The carpenter's busy schedule after March suggests that continued performance after the delay would be materially burdensome and thus his obligation should be completely discharged.

The owner of a milk container manufacturing firm sought to focus public attention on the milk packaging law of a certain state in order to have it repealed. On a weekday at 12:00 noon, he delivered an excited, animated, and loud harangue on the steps of the State Capitol in front of the main entryway. An audience of 200 onlookers, who gathered on the steps, heckled him and laughed as he delivered his tirade. The owner of the manufacturing firm repeatedly stated, gesturing expressively and making faces, that "the g-ddamned milk packaging law is stupid," and that "I will strangle every one of those g-ddamned legislators I can get hold of because this law they created proves they are all too dumb to live." After about fifteen minutes the owner of the manufacturing firm stopped speaking, and the amused crowd dispersed. There is one relevant statute of the state which prohibits "all speech making, picketing, and public gatherings of every sort on the Capitol steps in front of the main entryway between 7:45 a.m.-8:15 a.m., 11:45 a.m.-12:15 p.m., 12:45 p.m.-1:15 p.m., and 4:45 p.m.-5:15 p.m. on Capitol working days." If the owner of the milk container manufacturing firm is prosecuted under the "Capitol Steps" statute and defends on constitutional grounds, which of the following best describes the proper burden of proof?

A. The challenger would have to prove that the state did not have a rational basis for enacting this statute. B. The challenger would have to prove that the state did not have a compelling need for this statute or that it had less restrictive means by which it could satisfy that need. C. The state would have to prove that it had a rational basis for enacting this statute. D. The state would have to prove that it had an important government interest for enacting this statute and that the means by which the statute satisfied that interest were narrowly tailored. Answer: D. Generally, the content-based regulation of speech is subject to strict scrutiny. However, where a statute regulates conduct incidental to speech, the Court has allowed the government to adopt content-neutral, time, place, and manner regulations. If the regulation involves a public forum, it will be upheld if it is narrowly tailored to achieve an important government interest. Here, the statute is a content-neutral, time, place, and manner regulation involving a public forum. Therefore, answer D correctly states the proper test and places the burden of proof on the correct party.

A woman owned 400 acres of land, half of which was densely wooded and the other half of which was almost entirely occupied by a large gravel pit which supplied gravel for her small landscaping business. A small house located on the edge of the pit was in very poor condition and had been vacant for many years. The woman transferred the 400 acres to her son for life, with the remainder going to a local charity on the son's death. Now the son wants to increase the gravel production and expand the pit by tearing down the house. He also wants to cut the trees on the wooded half and sell them for profit. The local charity, holder of the remainder, sues to enjoin the son from doing any of these things. How will the court rule?

A. The charity can stop both the gravel mining and the tree cutting and can block the destruction of the house. B. The charity can stop neither the gravel mining nor the tree cutting but can block the destruction of the house. C. The charity can stop the tree cutting but not the gravel mining or the destruction of the house. D. The charity can stop the gravel mining and the tree cutting but not the destruction of the house because it is dangerous and unfit for use. Answer: C. The charity can stop only the tree cutting. The charity would be suing the life tenant on a theory of waste. Both the gravel mining and the tree cutting could be voluntary waste. Generally, a life tenant can only maintain the property and not sell off any of the natural resources, such as gravel and tress. But there there is an exception for existing exploitation of these resources. Because the gravel mine was operating prior to the son's taking of the life estate, that preexisting use is protected and the son is not liable for continuing the mining of gravel. The destruction of the house might be considered ameliorative waste - the destruction of improvements on the life estate that increase the value of the property. Generally, a life tenant cannot tear down improvements simply because the life tenant wants to make a more profitable use of the land. But an exception exists when changed conditions have made the destruction of the improvement reasonably necessary.

A man, a citizen of State A, is the class representative of a valid class action lawsuit in federal court in State B against a State B corporation. The lawsuit claims that the corporation breached the terms of its sales contract by failing to notify consumers after the corporation became aware of certain problems with its product. The class representative and the corporation agreed to settle the case, which would bind all members of the class and provide each member with coupons worth $250 towards the future purchase of any product sold by the corporation. With a class of 100,000 members, the coupons' total value is $25 million, and attorneys' fees were set at 10% ($2.5 million). Which of the following is true?

A. The class representative and corporation may, but need not, seek court approval of the settlement agreement. B. The court must allow absent class members to opt out of the settlement agreement. C. Attorneys' fees must be approved by the court, and this award will be approved. D. The court must hold a settlement hearing. Answer: D is correct. The CAFA includes a number of protections that apply to settlements in all class actions in federal court. One such protection concerns coupon settlements. Pursuant to the CAFA, a court may only approve a coupon settlement after the court holds a settlement hearing to ensure that the settlement is fair. A is incorrect. The Class Action Fairness Act ("CAFA") applies when (a) any class member is of diverse citizenship from any defendant, (2) the amount in controversy exceeds $5 million, and (3) there are at least 100 members in the proposed class. Here, the elements for CAFA subject matter jurisdiction are met. The CAFA includes a number of protections that apply to settlements in all class actions in federal court. One such protection concerns coupon settlements. Pursuant to the CAFA, a court may only approve a coupon settlement after the court holds a settlement hearing to ensure that the settlement is fair. Therefore, A is incorrect because it is not true that the parties need not seek court approval for the settlement. B is incorrect. The Federal Rules of Civil Procedure concerning class actions permit, but do not require the court to afford class members an opportunity to opt-out of the settlement before the court approves the settlement. See Fed.R.Civ.P. 23(e)(4). C is incorrect. CAFA provides that attorneys' fees may not be a percentage of the value of a coupon settlement; instead, fees may be calculated on the basis of hours worked or may be a percentage of the value of the coupons that are redeemed.

A plumber filed a jurisdictionally valid diversity lawsuit against a dentist in federal court in State A, which applies traditional contributory negligence principles, seeking compensation for damages incurred in a car accident. The dentist defended on the ground of contributory negligence. After a four-day trial, the judge instructed the jury to issue a verdict and also to answer two written questions: (1) Was the dentist negligent and (2) was the plumber contributorily negligent? The jury answered "yes" to both and returned a verdict in favor of the plumber, awarding him damages of $95,000. None of the parties filed any post-verdict motions. Which of the following is true?

A. The court must enter the jury's judgment. B. The court may enter the jury's judgment. C. The court may enter judgment for the dentist. D. The court must order a new trial. Answer: C is correct. The court may enter a judgment for the dentist. Here, the jury has entered an erroneous verdict due to its inconsistent determinations. When this happens, Federal Rule 49(b)(3) gives the court the choice among only three options: (1) to approve an appropriate judgment according to the answers, notwithstanding the verdict, (2) direct the jury to further consider its answers and verdict, or (3) order a new trial. Thus, A is incorrect because under these facts, the court does not have to enter the jury's judgment.

Assume that Congress passed and the President of the U.S. signed a law prohibiting individuals (other than police, the U.S. military, and other specified categories of persons) from carrying a firearm into any building or other enclosed space in which more than 25 people are present. A citizen of the U.S. was arrested and charged with violation of the federal firearms law. He sued the President in federal court, citing the language of the Second Amendment that "the right of the people to keep and bear Arms, shall not be infringed." The citizen sought money damages for violation of his Second Amendment rights and an injunction against future enforcement of the federal firearms law. The President moved to dismiss the claim, arguing that there is no private right of action, directly under the Second Amendment, for violation of Second Amendment rights. How should the district court rule on the motion to dismiss?

A. The court should grant the motion to dismiss because the court lacks subject matter jurisdiction over the case. B. The court should deny the motion to dismiss because the court has subject matter jurisdiction over the case. C. The court should grant the motion to dismiss for lack of jurisdiction if it concludes that there is indeed no private right of acton directly under the Second Amendment. D. The court should deny the motion to dismiss for lack of jurisdiction but dismiss for failure to state a claim upon which relief can be granted if it concludes that there is indeed no private right of action directly under the Second Amendment. Answer: D. When a plaintiff sues, purportedly under federal law, the federal courts have jurisdiction unless the claim is frivolous; there is nothing in the facts that suggest this claim is frivolous. However, even if the court has subject matter jurisdiction, that is not a sufficient reason not to dismiss. The court should still dismiss this action for failure to state a claim upon which relief can be granted if it concludes that there that there is no private right of action directly under the Second Amendment.

On March 1, a mechanic agreed to repair an owner's machine for $5,000, to be paid on completion of the work. On March 15, before the work was completed, the mechanic sent a letter to the owner with a copy to the mechanic's creditor, telling the owner to pay $5,000 to the creditor. The mechanic then completed the work. Which of the following, if true, would best serve the owner as a defense in an action brought against him by the creditor for $5,000?

A. The creditor was incapable of performing the mechanic's work. B. The mechanic had not performed his work in a workmanlike manner. C. On March 1, the mechanic had promised the owner that he would not assign the contract. D. The creditor was not the intended beneficiary of the mechanic-owner contract. Answer: Answer B is correct. The mechanic was not entitled to payment unless he satisfied his contractual duties. Every contract for work or services contains an implied duty to perform in good faith or in a workmanlike manner. The mechanic would still have to perform his obligations in good faith and in a workmanlike manner for owner to be obligated to pay. Payment then may be made to the creditor because the mechanic had assigned his rights under the contract to the creditor.

A woman owned two adjacent parcels. The east parcel fronts on a poor unpaved public road, while the west parcel fronts on a major highway. Fifteen years ago, the woman conveyed the east parcel to her son "together with a right-of-way 25 feet wide over the north side of the west parcel to the highway." At that time, the east parcel was improved with a 10-unit motel. Ten years ago, the woman died. Her will devised the west parcel "to my son for life, remainder to my daughter." Five years ago, the son executed a deed purporting to convey the east and west parcels to his friend in fee simple. The friend then enlarged the motel to 12 units. Six months ago, the son died and the daughter took possession of the west parcel. She brought an appropriate action to enjoin the friend from using the right-of-way. Who should prevail?

A. The daughter, because merger extinguished the easement. B. The daughter, because her friend has overburdened the easement. C. The friend, because he has an easement by necessity. D. The friend, because he has the easement granted by the woman to her son. Answer: D. The friend should prevail. The deed from the woman to her son granted the son an express easement. That easement was never terminated by merger and the friend, the owner of the motel property on the east parcel, still has a valid easement over the daughter's property, the west parcel.

The dealer in oriental rugs acquired an antique rug measuring 24 feet by 36 feet. A banker inspected the rug and orally agreed to buy it for the asking price of $65,000, provided he was successful in purchasing the house he was trying to buy, because it had a living room large enough to accommodate the rug. The sale agreement was later reduced to writing, but the provision concerning the purchase of the house was not included int he written agreement. If the banker is unsuccessful in acquiring the house he wants because the owner decided not to sell, and the dealer sues the banker for the purchase price, what is the most likely result?

A. The dealer will prevail because the original oral agreement need not be in writing to be enforceable. B. The dealer will prevail because of the parol evidence rule. C. The banker will prevail because he was unable to acquire the house he wanted. D. The banker will prevail because the dealer is not entitled to specific performance. Answer: C. The banker will prevail because he could not acquire the house. In general, the pER bars oral evidence contradicting a written agreement which was intended to be a final and exclusive embodiment of the parties' agreement. However, one exception to this general rule provides that parol evidence is admissible to show a condition precedent to the existence of a contract.

By warranty deed, a woman conveyed Blackacre to her friend and her neighbor "as joint tenants with right of survivorship." The friend and neighbor are not related. The friend conveyed all her interest to her boyfriend by warranty deed and subsequently died intestate. Thereafter, the neighbor conveyed to his girlfriend by warranty deed. There is no applicable statute, and the jurisdiction recognizes the common-law joint tenancy. Title to Blackacre is in

A. The girlfriend B. The woman C. The girlfriend and the boyfriend D. The grilfriend and the heirs of the friend. Answer: C. The facts state that the property was held by the friend and neighbor in joint tenancy. Where one joint tenant makes an inter vivos conveyance of their interest in property, a severance of the joint tenancy occurs and the interest transferred is that of a tenant in common. As soon as the friend made an inter vivos conveyance to her boyfriend, the joint tenancy was severed, and he and the neighbor became tenants in common. As such, the neighbor's conveyance to his girlfriend was likewise a transfer of an interest as a tenant in common. Therefore, the girlfriend and boyfriend hold title as tenants in common.

A homeowner from State A sued a man from State B in a federal court in State B. The single-count complaint alleged breach of contract, asked for specific performance, and demanded a jury under the Seventh Amendment. The man's lawyers argue that the homeowner is not entitled to a jury. How will the judge rule?

A. The homeowner is not entitled to a jury because the federal court's jurisdiction is based on diversity, and the Seventh Amendment applies only to federal question jurisdiction. B. The homeowner is entitled to a jury because this is a civil case in which the plaintiff seeks purely legal remedies. C. The homeowner is not entitled to a jury because this is a civil case in which the plaintiff seeks purely equitable remedies. D. The homeowner is entitled to a jury because this is a civil case in which the plaintiff seeks both legal and equitable remedies Answer: C is correct. The Seventh Amendment guarantees a jury for suits at common law, and specific performance is a remedy at equity, not law. A is incorrect. The Seventh Amendment's application does not depend on the basis for the court's jurisdiction. B is incorrect. The Seventh Amendment guarantees a jury for suits at common law, and specific performance is a remedy at equity, not law. D is incorrect. While it is true that the Seventh Amendment's jury guarantee generally is available where a plaintiff seeks both legal and equitable remedies, the plaintiff here seeks an exclusively equitable remedy.

A tenant entered into a two-year apartment lease with a landlord on July 1. Rent was specified in the lease to be $850 per month, payable on the first of each month. On June 15, near the end of the two year term, the landlord asked the tenant if he wanted to renew the lease for an additional term. The tenant said he would like to think about it, and the landlord agreed, but added that the rent for the new lease would be $975 per month. The landlord heard nothing form the tenant. On July 10, the landlord found that the tenant was still in the apartment and told him that he was imposing a new tenancy on the tenant for the period allowed by the law under these circumstances. What tenancy can be imposed, and at what rent?

A. The landlord can impose a new periodic tenancy of month-to-month and the rent will be $850 per month. B. The landlord can impose a new periodic tenancy of month-to-month and the rent will be $975 per month. C. The landlord can impose a new periodic tenancy of year-to-year and the rent will be $850 per month. D. The landlord can impose a new periodic tenancy of year-to-year and the rent will be $975 per month. Answer: B. The tenancy will be month-to-month at $975 per month. When a tenant wrongfully holds over after the expiration of a lease, the landlord has two choices: either treat the tenant as a trespasser and sue for damages and possession or impose a new periodic tenancy on the hold-over tenant. If the landlord chooses, as this landlord has done here, to impose the new periodic tenancy, most courts in residential situations would impose a month-to-month tenancy. While the rent (as well as other terms) of the new tenancy will generally be the same as the old tenancy, there is an exception when the landlord has told the tenant of a future higher rent and that notification came before the expiration of the old lease. In that event, the landlord can impose the higher rent in the new periodic tenancy.

Two salesmen, who lived in different suburbs twenty miles apart, were golfing acquaintances at the Interurban Country Club. Both were traveling salesmen--one for a pharmaceutical company and the other for a widget manufacturer. The pharmaceutical salesman wrote the widget salesman by United States mail on Friday, October 8: I need a motorcycle for transportation to the country club and will buy your Sujocki for $1,200 upon your bringing it to my home address above [stated in the letterhead] on or before noon, November 12 next. This offer is not subject to countermand. Sincerely, [signed] the pharmaceutical salesman The widget salesman replied by mail the following day: I accept your offer, and promise to deliver the bike as you specified. Sincerely, [signed] the widget salesman This letter, although properly addressed, was misdirected by the postal service and not received by the pharmaceutical salesman until November 10. The pharmaceutical salesman had bought another Sujocki bike from a different friend for $1,050 a few hours before. The friend saw the widget salesman at the Interurban Country Club on November 11 and said: "I sold my Sujocki to the pharmaceutical salesman yesterday for $1,050. Would you consider selling me yours for $950?" The widget salesman replied: "I'll let you know in a few days." On November 12, the widget salesman took his Sujocki to the pharmaceutical salesman's residence; he arrived at 11:15 a.m. The pharmaceutical salesman was asleep and did not answer the ringing doorbell until 12:15 p.m. The pharmaceutical salesman then rejected the widget salesman's bike on the ground that he had already bought someone else's bike. In a lawsuit by the widget salesman against the pharmaceutical salesman for breach of contract, what would the court probably decide regarding the widget salesman's letter of October 9?

A. The letter bound both parties to a unilateral contract as soon as the widget salesman mailed it. B. Mailing of the letter by the widget salesman did not, of itself, prevent a subsequent, effective revocation by the pharmaceutical salesman of his offer. C. The letter bound both parties to a bilateral contract, but only when received by the pharmaceutical salesman on November 10. D. Regardless of whether the pharmaceutical salesman's offer had proposed a unilateral or a bilateral contract, the letter was an effective acceptance upon receipt, if not upon dispatch. Answer: Answer B is correct. The widget salesman's letter was not an appropriate method of acceptance because the pharmaceutical salesman's letter offer requested performance; thus, the mail box rule would not apply to make the acceptance effective upon dispatch, and the pharmaceutical salesman's offer remained freely revocable. A is incorrect because the letter could not be reasonably construed as the beginning of performance. C is incorrect because the offer called for a unilateral acceptance. D is also incorrect because the offer did not invite the widget salesman to choose between acceptance by promise and acceptance by performance.

The owner of land conveyed it "to my best friend, and upon her death to my daughter." The best friend took possession of the land and lives there for four years. She then conveyed "my interest in land" to her longtime neighbor. Since the neighbor took up residence on the land, he has been sent two county property tax bills, which he has refused to pay. The county is now threatening to force a sale of the property to satisfy the tax lien. The daughter filed an appropriate suit, asking the court to evict the neighbor from the land and to compel him to pay the taxes for his period of occupancy. How should the court rule?

A. The neighbor has a life estate in the land and he must pay the taxes on the property. B. The neighbor has a life estate in the land for the period of the friend's life and the neighbor does not have to pay the taxes on the property because taxes are the responsibility of the holder of the future interest. C. The neighbor has a life estate in the land for the period of the friend's life and the neighbor must pay the taxes on the property. D. The daughter owns the land, because the friend could not convey her interest to her neighbor since the daughter held the future interest. Answer: C. The neighbor has a life estate for the life of the friend and must pay taxes. When the best friend transferred her life estate to the neighbor, he took an estate that will terminate when the best friend dies. That is all that the best friend owned and all that she could convey. In addition, the rule is that the life tenant is responsible for paying taxes on the property (had there been a mortgage, the life tenant would have had to pay the interest on the mortgage but not the principal.

A patient, a citizen of State A, brings a tort claim based on diversity against a cardiologist, located in State B, in State A federal court. Federal Rules of Civil Procedure state that "[a] civil action is commenced by filing a complaint with the court." Tort claims have a three year statute of limitations under State A law, and a State A statute provides that statutes of limitations are tolled when service of process is made on the defendant. The patient's tort claim arose on January 5, 2005. The patient filed his lawsuit with the court on January 5, 2008, but the cardiologist was not served until January 15, 2008. Which of the following is true?

A. The patient's claim was not timely filed because it did not comply with State A's tolling statute. B. The patient's claim was timely because the Federal Rules of Civil Procedure, not state law, apply and the action was "commenced" on January 5, 2008. C. The patient's claim is time-barred because the Federal Rules of Civil Procedure are not applicable because this is a diversity action. D. The patient's claim is not time-barred; State A law is not applicable because this is a diversity action, and the patient's lawsuit is timely under the Federal Rules of Civil Procedure. Answer: A is correct. Under the Erie doctrine, in a diversity action, a federal court must apply the substantive law of the state. The Supreme Court has established that statutes of limitations and rules for tolling statutes of limitations are substantive for Erie doctrine purposes. Therefore, in this case, State A law applies.

A first-time home buyer financed the purchase of a house with a $100,000 mortgage she took out with a bank. The mortgage was recorded. A few years later she borrowed $5,000 from a finance company to pay for a foreign trip, using her house as security. The finance company promptly and properly recorded its mortgage on the property. One year after that, she borrowed $40,000 form an equity company to pay for an addition on the house. The equity company promptly and properly recorded the mortgage it took on the property. Shortly thereafter, she lost her job and was unable to make payments on either the finance company's or the equity company's mortgages, but she was able to make payments on the bank's mortgage. The finance company filed foreclosure of its mortgage and included the equity company in the action, and a purchaser bought the property at the foreclosure sale. What is the purchaser's obligation regarding the bank's mortgage and the equity company's mortgage?

A. The purchaser takes the property subject to both mortgages. B. The purchaser takes the property subject to neither mortgage. C. The purchaser takes the property subject to the equity company's mortgage, but not subject to the bank's mortgage. D. The purchaser takes the property subject to the bank's mortgage, but not subject to the equity company's mortgage. Answer: D. The purchaser takes the property subject only to the bank's mortgage. A foreclosure sale wipes out all junior mortgages (those that come later in time than the mortgage that was foreclosed) but doe snot wipe out senior mortgages (those that came earlier). Because the bank's mortgage preceded the finance company's it is senior and is not wiped out. The purchaser takes subject to this mortgage.

An elderly grandfather who wanted to ensure that his property would remain in the family after his death included the following clause in his will: "I give my house in the city to my son, but if he ever tries to sell it while he is alive, I want it taken away from him and given to my grandson." The grandfather's will was properly executed. When the grandfather later died, what interests did the son and grandson take in the property?

A. The son took a fee simple. B. The grandson took a fee simple. C. The son took a fee simple subject to an executory interest, and the grandson took an executory interest. D. The son took a fee simple determinable, and the grandson took a contingent remainder. Answer. A. The son received a fee simple in the property. The grandfather attempted to give his son a fee simple, but placed a restrain on alienation. Direct restraints on alienation of a fee simple are void.

The defendant and an accomplice were committing an armed robbery of a convenience store when the police arrived. A shootout ensued, resulting in the death of someone on the scene. The defendant is arrested and charged with felony murder in a jurisdiction that follows the agency theory of liability for felony murder. The defendant is most likely to be found guilty if the victim was which of the following?

A. The store clerk shot by the defendant's accomplice after teh clerk had pulled out a gun from under the counter. B. The store clerk shot by the police, who were shooting at the defendant. C. The defendant's accomplice shot by the police during the shootout. D. A customer shot by the store clerk, who was shooting at the defendant. Answer: A. The defendant wil be most likely guilty of felony murder for the shooting by his accomplice. Under the felony murder doctrine, a killing committed during the course of a felony is murder. Under conspiracy and accomplice liability law, all participants in the felony will be liable for murder if the killing was foreseeable. Resistance by the store clerk was foreseeable and neither the accomplice nor the defendant had any right to self-defense under the circumstances. Hence, the defendant is liable for the shooting of the store clerk by his accomplice during the armed robbery.

A horse jockey from State A filed suit against an animal trainer from State B in federal court in State B seeking $80,000 as compensation for damages to his race horse as a result of the trainer's alleged negligence. The horse jockey mailed to the trainer a request for waiver of service, two copies of the waiver form, and a copy of the horse jockey's complaint. Which of the following is true?

A. The trainer may waive service but would not be penalized for failing to do so even absent a showing of good cause for failure to waive. B. The trainer should do nothing because there is no procedure to properly waive service. C. The trainer must waive service or will face penalties from the court absent a showing of good cause for failure to waive. D. State law determines whether waiver is permissible since this is a diversity action. Answer: C. The plaintiff may request the defendant to waive service of process. To request a waiver of service, the plaintiff must mail the defendant a formal request to waive service, two copies of the waiver form, and a copy of the complaint. The defendant has 30 days from the date that the request was sent to return the waiver. If the defendant does not waive service of process, the plaintiff must serve him using one of the other approved methods (e.g., personal service, service at the defendant's usual abode, service upon an authorized agent, or service under a state approved method). However, the defendant will be liable for the cost of such additional service if he does not have good cause for failing to waiver service. Here, the horse jockey followed the correct protocol for waiver of service. Therefore, absent a showing of good cause, the trainer must waive service or face the court imposed penalties, such as costs of the additional service methods.

On March 1, a seller orally agreed to sell his land, Homestead, to a buyer for $46,000 to be paid on March 31. The buyer orally agreed to pay $25,000 of the purchase price to a creditor of the seller in satisfaction of a debt which the seller said he had promised to pay the creditor. On March 10, the buyer dictated the agreement to his secretary but omitted all reference to the payment of the $25,000 to the creditor. In typing the agreement, the secretary mistakenly typed $45,000 rather than $46,000 as the purchase price. Neither the buyer nor the seller carefully read the writing before signing it on March 15. Neither noticed the error in price and neither raised any question concerning omission of the payment to the creditor. In an action by the creditor against the buyer for $25,000, which of the following, if proved, would best serve the buyer as a defense?

A. There was no consideration to support the seller's antecedent promise to pay the creditor the $25,000. B. On March 5, before the creditor was aware of the oral agreement between the seller and the buyer, the seller agreed with the buyer not to pay any part of the purchase price to the creditor. C. Whatever action the creditor may have had against the buyer was barred by the statute of limitations prior to March 1. D. Before he instituted his action against the buyer, the creditor had not notified either the buyer or seller that he had accepted the buyer-seller arrangement for paying the creditor. Answer: Answer B is correct. A third party acquires standing to enforce a promise only if that party is an intended beneficiary of the promise. The promisor and promisee retain the power to modify or discharge a duty owed to an intended beneficiary until the beneficiary's rights vest. The beneficiary's rights will vest if he manifests assent to or materially changes his position in justifiable reliance on the promise before receiving notice of the discharge. Since the promise at issue is the buyer's promise to pay $25,000 "to the creditor," the creditor is an intended beneficiary because the performance flows directly to him. However, if the buyer and seller later agree not to pay any of the purchase price to the creditor before the creditor was aware of the contract, that agreement would be effective in discharging the creditor's rights as an intended beneficiary because those rights would not have vested.

A seller agreed to sell a tract of land by an enforceable written contract to a buyer for an agreed price. At the time and place designated for closing, the seller tendered the deed to the land, but the buyer refused to complete the transaction because of a secured note on the land, saying that the contract required the seller to supply a deed free of encumbrances. The seller responded that she intended to pay the secured loan, which was substantially less than the contract price, with the proceeds of the sale of the land. The seller offered to put the proceeds of the sale in escrow for that purpose, for which the buyer could pick the escrow agent. The buyer refused the seller's proposal. The seller then brought suit against the buyer for specific performance of the land sale contract. What is the seller's best argument in support of her claim for relief?

A. Under the doctrine of equitable conversion, title has already passed to the buyer and the only issue is how the purchase price is to be allocated. B. As the seller of real estate, the seller had an implied right to use the proceeds to clear the title being conveyed. C. Because the seller partially performed, the agreement is specifically enforceable. D. A secured note does not remain attached to property, but rather remains attached to the borrower. Answer: B. The seller had an implied right to use the proceeds to clear title. It is true that the seller is obligated to transfer a title free of encumbrances, and a mortgage qualifies as an encumbrance. It is also true, however, that a seller may use proceeds paid by the buyer at closing to pay off mortgages, and thus deliver clear title to the buyer. As long as the amount due from the buyer is enough to pay off the note, the seller has an implied right to do this, an the existence of the note under these circumstances doe snot breach the contract sale.

In one state, certain advanced diagnostic medical technologies were located only in hospitals, where they provided a major source of revenue. In many other states, such technologies were also available at "diagnostic centers" that were not affiliated with hospitals. A group of physicians announced its plan to immediately open in the state a diagnostic center that would not be affiliated with a hospital. The state hospital association argued to the state legislature that only hospitals could reliably handle advanced medical technologies. The legislature then enacted a law prohibiting the operation in the state of diagnostic centers that were not affiliated with hospitals. The group of physicians filed suit challenging the constitutionality of the state law. What action should the court take?

A. Uphold the law, because the provision of medical services is traditionally a matter of legitimate local concern that states have unreviewable authority to regulate. B. Uphold the law, because the legislature could rationally believe that diagnostic centers not affiliated with hospitals would be less reliable than hospitals. C. Invalidate the law, because it imposes an undue burden on access to medical services in the state. D. Dismiss the suit without reaching the merits, because the suit is not ripe. Answer: B. The law does not trigger heightened judicial scrutiny because it neither classifies regulatory subjects on a constitutionally suspect basis nor unduly burdens the exercise of a fundamental right. The appropriate constitutional standard of review therefore is whether the law is rationally related to a legitimate government interest. The apparent legislative judgment that diagnostic centers not affiliated with hospitals would be less reliable than hospitals is rational, regardless of whether it is in fact correct. Thus, Answer B is correct.

In 1970, a cattle company paid $30,000 for a 150-acre tract of agricultural land well suited for a cattle feed lot. The tract was ten miles from the city and five miles from the nearest home. By 2006, the city limits extended to the cattle company's feed lot. About 10,000 people lived within three miles of the cattle-feeding operation. The cattle company land is outside the city limits and no zoning ordinance applies. The cattle company land is now worth $300,000, and $25,000 has been invested in buildings and pens. The cattle company uses the best and most sanitary feed lot procedures, including chemical sprays, to keep down flies and odors and frequently removes manure. Despite these measures, residents of the city complain of flies and odors. An action has been filed by five individual homeowners who live within half a mile of the cattle company feed lot. The plaintiffs' homes are valued currently at $25,000 to $40,000 each. Flies in the area are five to ten times more numerous than in other parts of the city, and extremely obnoxious odors are frequently carried by the wind to the plaintiffs' homes. The flies and odors are a substantial health hazard. If plaintiffs assert a claim based on nuisance, plaintiffs will

A. prevail, because the cattle company's activity unreasonably interferes with plaintiffs' use and enjoyment of their property. B. prevail, because the cattle company's activity constitutes an inverse condemnation of their property. C. not prevail, because the cattle company had operated the feed lot for more than 25 years. D. not prevail, because the cattle company uses the most reasonable procedures to keep down flies and odors. Answer: A is the correct answer. A private nuisance is substantial and unreasonable interference with a possessor's use or enjoyment of his property. Interference is substantial if it is offensive to an average person in the community. Interference is unreasonable if the injury to the plaintiff outweighs the utility of the defendant's conduct. In making this determination, the court will consider the neighborhood, land value, and alternative courses of conduct for the defendant. In this situation, the interference is unreasonable because the injury outweighs the utility of the feed lot. Due to the growth of the city, there are now a substantial number of residences near the feed lot. Furthermore, while the cattle company's land is worth $300,000, the company has only invested $25,000 in the establishment of the feed lot, and is severely impairing the use of between $125,000 to $200,000 worth of residential property. Therefore, the interference will be considered a private nuisance. B is incorrect. Inverse condemnation is a property action brought by the owner against a government entity that has taken the owner's property without the use of formal condemnation proceedings. The cattle company is not a government entity. C is incorrect. Generally, if the purchaser bought in good faith, the purchaser is entitled to the reasonable use or enjoyment of the land. Therefore, coming to the nuisance does not automatically preclude a nuisance action. D is incorrect. The defendant's use of reasonable procedures is an insufficient defense if the flies and odors are still a substantial and unreasonable interference with the plaintiffs' use or enjoyment of their property.

As a result of an accident at the NPP nuclear power plant, a quantity of radioactive vapor escaped from the facility and two members of the public were exposed to excessive doses of radiation. According to qualified medical opinion, that exposure will double the chance that these two persons will ultimately develop cancer. However, any cancer that might be caused by this exposure will not be detectable for at least ten years. If the two exposed persons do develop cancer, it will not be possible to determine whether it was caused by this exposure or would have developed in any event. If the exposed persons assert a claim for damages against NPP shortly after the escape of the radiation, which of the following questions will NOT present a substantial issue?

A. Will the court recognize that the plaintiffs have suffered a present legal injury? B. Can the plaintiffs prove the amount of their damages? C. Can the plaintiffs prove that any harm they may suffer was caused by this exposure? D. Can the plaintiffs prevail without presenting evidence of specific negligence on the part of NPP? Answer: D is the correct answer. The call of the question wants you to find the answer that is NOT a substantial issue. The operation of a nuclear power that contains highly toxic chemicals/ radioactive vapor is considered an abnormally dangerous activity. Ultra-hazardous activities give rise to strict liability because the inherent danger or peculiar risk is unreasonably high when compared to its social utility, even in the absence of negligence and where all the proper precautions have been taken. Evidence of negligence, therefore, is not an issue in this situation, which makes D the best answer. A is a substantial issue. The plaintiff's only current harm is increased risk of future injury, which is not universally recognized as sufficiently concrete to grant standing. B is a substantial issue because the damages have not yet manifested and actual damage is only a possibility. C is a substantial issue because for strict liability to apply, the plaintiffs generally must show that the defendant's emissions caused the injury.

On March 1, a computer programming company orally agreed with a department store to write a set of programs for the department store's computer and to coordinate the programs with the department store's billing methods. A subsequent memo, signed by both parties, provided in its entirety: The department store will pay the computer programming company $20,000 in two equal installments within one month of completion if the computer programming company is successful in shortening by one-half the processing time for the financial transactions now handled on the department store's Zenon 747 computer; the computer programming company to complete by July 1. This agreement may be amended only by a signed writing. On June 6, the computer programming company demanded $10,000, saying the job was one-half done. After the department store denied liability, the parties orally agreed that the department store should deposit $20,000 in escrow, pending completion to the satisfaction of the department store's computer systems manager. On July 5, the computer programming company completed the programs. Tests showed that the computer programs cut processing time by only 47 percent. The department store's computer systems manager refused in good faith to certify satisfactory completion. The department store requested the escrow agent to return the $20,000 and asserted that nothing was owed to the computer programming company even though the department store continued to use the programs. If the computer programming company in fact had half-completed the job on June 6, would it then have been entitled to $10,000?

A. Yes, because June 6 was within one month of completion B. Yes, because the computer programming company had done one-half the job. C. No, because of a constructive condition precedent requiring at least substantial completion of the work before the department store would have a duty to pay. D. No, because "within one month of completion" would, in these circumstances, be interpreted to mean "within one month after completion." Answer: Answer D is correct. The rules of contract construction dictate that the courts will construe words according to their ordinary meaning unless it is clear that they were meant to be used in a technical sense. While "within one month of completion" could be technically interpreted to refer to the month before the completion, there is no indication that these words were meant to be used in the technical sense. Therefore, the phrase will be given its ordinary meaning, that the payment will be received within one month after completion of the project.

A landlord leased a warehouse building and the lot on which it stood to a tenant for a term of ten years. The lease contained a clause prohibiting the tenant from subletting his interest. Can the tenant assign his interest under the lease?

A. Yes, because restraints on alienation of land are strictly construed. B. Yes, because disabling restraints on alienation of land are invalid. C. No, because the term "subletting" includes "assignment" when the term is employed in a lease. D. No, because even in the absence of an express prohibition on assignment, a tenant may not assign without the landlord's permission. Answer: A. This question requires you to distinguish between an assignment and a sublease. An assignment of a lease by a tenant occurs where the tenant transfers all his rights and duties under a lease to another party (an "assignee") for the entire length of time remaining on the lease. By contrast, a sublease is a transfer by a tenant to another party (a "sublessee") for a time period shorter than the time remaining on the lease. A landlord is free to place limits on alienation within the terms of the lease, thereby preventing the tenant from reassigning, subletting, or both. Any such restriction, however, will be strictly construed according to its explicit terms. In this case, the landlord has only prohibited the tenant from subletting. The restriction will be strictly construed, and since it does not state that assignment is prohibited, the tenant is free to assign his interest.

After a difficult divorce, a mother wrote to her son and daughter the following: In consideration of your emotional support for me during that trying time and your love and affection for me, I promise to divide my estate between you in equal shares. You know you can count on your mother's word. The daughter thereafter continued her usual practice of calling her mother once a week and visiting her at Christmas and on her birthday until her mother died three years later. Shortly after the funeral, the daughter learned that the mother's will made the son the sole legatee. If the daughter sues the executor of the mother's estate for one-half of that estate, based on the mother's letter to her, will she win?

A. Yes, because she relied to her detriment on her mother's promise by visiting her mother. B. Yes, because her emotional support of her mother during the divorce proceedings constituted valid consideration for her mother's promise. C. Yes, because even though her mother's letter is as promise to make a gift in the future, the promise is in writing and intended by her to be enforceable and therefore needs no consideration. D. No, because the mother's promise was not supported by consideration. Answer: D. The daughter will lose because there is no consideration to support the promise. Promises to make gifts in the future are unenforceable even if they are in writing an dare intended by the promisor to be enforceable.

Two men were arrested while riding in a stolen automobile. They were taken to the police station, booked, and fingerprinted. They were then taken to an interrogation room. After the detective gave them their Miranda warnings, one of the men said, "Forget it. As soon as you check for outstanding warrants, you'll find out that I escaped from prison. Since I am going back anyway, it's a farce to deny that we stole the car." The other man said nothing, and the first man proceeded to write and sign a full confession. The man who remained silent pled not guilty to the charge of grand theft auto. At his trial the prosector seeks to introduce evidence to show that he did not deny that he stole the automobile when the other man told the police in front of him that he was a party to the theft. Should the court hold that this evidence is proper?

A. Yes, because silence in this situation is indicative of guilt and is an implied admission. B. Yes, because the man who confessed had voluntarily waived his right to remain silent. C. No, because an accomplice's evidence is inherently unreliable and unduly prejudicial. D. No, because under this circumstance, the man who remained silent had no duty or responsibility to deny the allegations. Answer: D. The court should hold that this evidence is improper because the man who remained silent had no duty to deny the allegations here. A defendant in custody has no duty to speak at all, and the exercise of this constitutional right cannot be used against a defendant to show probable guilty. The Miranda warnings carry an implicit assurance that silence will carry no penalty.

A company operated an installation for distributing sand and gravel. The installation was adjacent to a residential area. On the company's grounds there was a chute with polished metal sides for loading sand and gravel into trucks. The trucks being loaded stopped on the public street below the chute. A completely effective method for securing the chute was available, but the company decided it was not worth the moderate cost. Instead, after closing hours, a plywood screen was placed in the chute and the ladder used for inspection was removed to another section of the installation. For several months, however, a number of children, 8 to 10 years of age, had been playing on the company's property and the adjoining street after closing hours and had figured out how to use the chute as a slide. The company knew of this activity. One evening, as children were playing on the chute, a commuter driving by the chute hit an 8-year-old boy who slid down in front of the automobile. The commuter applied her brakes, but they suddenly failed., and she hit and injured the child. The commuter saw the child in time to have avoided hitting him if her brakes had worked properly. Two days earlier, the commuter had taken her car to a mechanic to have her brakes inspected, and the mechanic had told her that the brakes were in perfect condition. Claims were asserted on behalf of the child by his proper legal representative against the company, the commuter, and the mechanic. With respect to the child's claim against the company, will the child prevail?

A. Yes, because the company could have effectively secured the chute at moderate cost. B. Yes, because the company is strictly liable for harm resulting from an artificial condition on its property. C. No, because the commuter had the last clear chance to avoid the injury. D. No, because the child was a trespasser. Answer: A. The issue here is the company's duty to known child trespassers (sometimes called the attractive nuisance doctrine). The company is subject to liability to the children for dangers caused by an artificial condition on its land because the company (1) knew that the children played on the chute by removing the plywood, (2) knew the children would slide down to the street below, (3) knew that the children would not realize that sliding into a street could be dangerous, and (4) the activity posed a high risk of harm to the children compared to the slight burden of eliminating the danger. A court would most likely find that the children were too young to appreciate the danger and that the company needed to exercise reasonable care to protect the children. Therefore, the child will prevail against the company in a negligence action if the child can prove by a preponderance of the evidence that there was an effective means of securing the chute at a moderate cost; for instance, by placing locks on the plywood screen. (Answer C - mistakes the rule of the law)

A company operates a factory that requires the use of very high voltage electricity. A neighbor owns property adjacent to the factory where he has attempted to carry on a business that requires the use of sensitive electronic equipment. Occasionally, the effectiveness of the neighbor's electronic equipment is slightly impaired by electrical interference arising from the high voltage currents used in the company's factory. The neighbor has complained to the company several times, with no result. There is no way that the company, by taking reasonable precautions, can avoid the interference with the neighbor's operation that arises from the high voltage currents necessary to the company's operation. In the neighbor's action against the company to recover damages for the economic loss caused to him by the electrical interference, will the neighbor prevail?

A. Yes, because the company's activity is abnormally dangerous. B. Yes, for loss suffered by the neighbor after the company was made aware of the harm its activity was causing to the neighbor. C. No, because the company did not cause a substantial and unreasonable interference with the neighbor's business. D. No, because the neighbor's harm was purely economic and did not arise from physical harm to his person or property. Answer: C is the correct answer. Private nuisance is defined as a condition or activity that interferes with a landowner's use and enjoyment of his land to such an extent that the landowner cannot reasonably be expected to bear the condition without compensation. The scope of interference is personal discomfort to the occupants or tangible harm to property, resulting in a diminution of its market value. Nuisance does not require proof of negligence; it just requires actual damages. In this case, the harm to the business was only occasional slight interference. Because the electrical interference was not a substantial and unreasonable interference with the neighbor's use and enjoyment of his property, he will not prevail. A is incorrect because there is no evidence that the activity cannot be performed without risk of serious harm to persons or property no matter how much care is exercised. B is incorrect. While it is true that nuisances generally must be intentional interferences, this interference is still not substantial nor unreasonable enough to be actionable. D is incorrect. Under a claim for nuisance, the neighbor could show that the electrical current caused physical harm to his equipment, thereby unreasonably damaging his business property interests with substantial electrical interference.

A father gave his daughter marketable title to a five-acre parcel of undeveloped land that adjoined 200 acres of uninhabited forest owned by a neighbor. When she visited her property with her father, he mistakenly pointed out the boundary line. She subsequently staked out the boundaries and built a log fence along what she thought was the boundary line. Approximately an acre of the neighbor's land was inside her fence. The daughter built a cabin and lived in it for 30 years until she had to sell the property for medical reasons. The daughter entered into a contract to sell the land. In accordance with the contract, the purchaser had a survey of the land done, which revealed the boundary discrepancy in the legal description. The purchaser contracted the neighbor, who said he knew nothing of the matter and did not consent to the daughter's placement of the fence on his property. The purchaser then refused to proceed with the purchase. The jurisdiction in which the parcel was located had a 20 year period of occupation to satisfy the requirements of adverse possession. If the daughter sues for specific performance of the land sale contract, will she prevail?

A. Yes, because the daughter satisfied all elements required to make out adverse possession of the portion of the neighbor's property within her fence. B. Yes, because the land sale contract was unaffected by the minor discrepancy in the legal description. C. No, because the daughter's title to the land is not marketable. D. No, because the daughter's fencing in of the neighbor's property was not hostile, even if the neighbor had no knowledge of her actions. Answer: C. The daughter will not prevail because her title is not marketable. The daughter did satisfy all six requirements of adverse possession of the portion of the neighbor's property. She possessed it exclusively for more than the statutory period, her possession was continuous and adverse to the rights of the true owner, and her possession was visible for all to see. But, even though she did acquire title by adverse possession, that title is not marketable until a judicial action is taken to quiet title. The law does not require purchasers to have to go to court to clearly establish title. Because the daughter had not done this, the title she contracted to convey was not, in fact, marketable.

An actess, who played the lead role in a television soap opera, was seriously injured in an automobile accident caused by teh defendant's negligent driving. As a consequence of the actress's injury, the television series was canceled, and a supporting actor was laid off. Although the supporting actor looked for other work, he remained unemployed. In an action against the defendant, can the supporting actor recover for his loss of income attributable to the accident?

A. Yes, because the defendant's negligence was the cause in fact of the supproitng actor's loss. B. Yes, because the supporting actor took reasonable measures to mitigate his loss. C. No, because the defendant had no reason to foresee that by injuring the lead actress he would cause harm to the supporting actor. D. No, because the defendant's liability does not extend to economic loss to the supporting actor that arises solely from physical harm to the lead actress. Answer: D. With the exception of a wrongful death claim allowed by statute, a negligence action for pure economic loss to a plaintiff as the result of an injury suffered by a third party is generally not recoverable.

A driver was stopped by the police after running a red light. Her roommate was also in the car. Because the driver did not have a driver's license, the officer lawfully placed her under arrest and put her in his squad car. At the time of the arrest, the officer saw a shopping bag in the back seat containing clothes with price tags on them. The officer asked the driver if she had made any other purchases that day, and she responded that there were additional purchases in the trunk. The officer then searched the trunk. The officer then searched the trunk of the car, where he found additional clothes purchases along with a clear plastic bag containing what appeared to be marijuana. Later testing confirmed that it was marijuana, which the roommates had purchased that morning from a neighbor. The two roommates were charged with possession of marijuana. Prior to her trial, the driver's attorney moved to suppress evidence of the marijuana because it was discovered in an illegal search, and the motion was granted. If the attorney for the other roommate who was the passenger in the car subsequently moves to suppress evidence of the marijuana at her trial, should her motion be granted?

A. Yes, because the marijuana was the fruit of an illegal search. B. Yes, because the judge had suppressed this evidence at the driver's trial. C. No, because she has no standing to object to an illegal search. D. No, if she admits that she owns the marijuana. Answer: C. The roommate's motion should be denied. The roommate has no standing to challenge the search of the trunk because the search did not violate her reasonable expectation of privacy. Merely being a passenger in someone else's car does not create a reasonable expectation of privacy with regard to a search of the car. Something more is needed to have standing to challenge the search, such as if the roommate owned the car (not indicted by the facts).

A sporting goods shop was burglarized by an escaped inmate from a nearby prison. The inmate stole a rifle and bullets from a locked cabinet. The burglar alarm at the shop did not go off because the shop's owner had negligently forgotten to activate the alarm's motion detector. Shortly thereafter, the inmate used the rifle ammunition stolen from the shop in a shooting spree that caused injury to several people, including the plaintiff. If the plaintiff sues the shop's owner for the injury she suffered, will the plaintiff prevail?

A. Yes, because the plaintiff's injury could have been prevented had the motion detector been activated. B. Yes, because the shop's owner was negligent in failing to activate the motion detector. C. No, because the storage and sale of firearms and ammunition is not an abnormally dangerous activity. D. No, because there is no evidence of circumstances suggesting a high risk of theft and criminal use of firearms stocked by the shop's owner. Answer: D is the correct answer. The commission of a criminal act often supersedes the liability of the original negligent actor. An exception occurs if the negligent act creates a condition such that a criminal act is the foreseeable consequence of that action. The shop owner didn't forget to lock his door; he just forgot to set the alarm on his gun shop. Because there is no evidence that the store is in a high crime area or that theft is common, D is the best answer. A is incorrect because the store owner did not have a duty to prevent the criminal acts of another unless those criminal acts were foreseeable. B is incorrect because the commission of the criminal act will supersede the store owner's liability. C does not address the appropriate theory of recovery. This is an issue of negligence, not strict liability.

A shoe retailer ordered by telephone 12 pairs of dress shoes form a shoe manufacturer at its list price of $500 per dozen; delivery in 20 days. The manufacturer orally accepted the offer, and immediately faxed to the retailer this signed memo: Confirming our agreement today for your purchase of a dozen pairs of dress shoes for $500, the shipment will be delivered in 20 days." Although the retailer received and read the manufacturer's message, he rejected the conforming shipment when it timely arrived. On learning of the rejection, does the manufacturer have a cause of action against the retailer for breach of contract?

A. Yes, because the shoes were identified to the contract and tendered to the retailer. B. Yes, because the manufacturer's faxed memo to the retailer was sufficient to make the agreement enforceable. C. No, because the agreed price was $500 and the retailer never signed a writing evidencing a contract with the manufacturer. D. No, because the retailer neither paid for nor accepted any of the goods tendered. Answer. B. The manufacturer has a cause of action for breach of contract. Because the contract was for goods priced at $500, the SOF must be satisfied under UCC 2-201. Because both the manufacturer and the retailer are merchants, a memorandum of the terms of the sale sent by the manufacturer to the retailer satisfies the SOF unless the retailer objects within 10 days, which she did not do.

A shopkeeper loaned a long-time employee $1,500 form his personal bank account because a family illness was causing the employee unexpected financial difficulties. Because the employee had proved himself to be trustworthy there was no writing evidencing the loan and no payback date established; it was understood that the employee would repay the loan when he was able to do so. Sometime later, the shopkeeper's nephew asked him if he could help fund a business that he was starting up. Because most of the shopkeeper's assets were currently tied up, he asked his employee if he would be in a position to repay the $1,500 form the employee on the following Monday. When Monday came, the employee decided he would rather tender the money to the shopkeeper than to someone he did not know, and the shopkeeper accepted the money. If the nephew never receives any money from the shopkeeper, will he succeed in an action against the employee for the $1,500.

A. Yes, because the shopkeeper effectively assigned his right to collect the $1,500 to the nephew. B. No, because the shopkeeper's acceptance of the $1,500 from teh employee revoked the shopkeeper's gift to the nephew. C. No, because the assignment was unsupported by consideration and therefore never effective. D. No, because the employee's tender of the $1,500 to the shopkeeper and the shopkeeper's acceptance for it constituted a novation. Answer: B. The Shopkeeper validly assigned his rights to receive the money to his nephew. However, this assignment was revocable, and it was revoked when the shopkeeper accepted the money from the employee. A creditor's right to receive money due from a debtor is a right that can be assigned, regardless of whether the debt is evidenced by a writing. By telling the employee to pay the money tot he nephew, the shopkeeper manifested an intent to transfer his right completely and immediately to the nephew. Neither a writing nor consideration was required for this assignment to be valid. However, these factors do not affect revocability. An exception to this rule arises when the assignor is estopped from revoking because he should reasonably foresee that the assignee will change his position in reliance on the assignment and such detrimental reliance occurs.

A wealthy widow, wishing to make a substantial and potentially enduring gift to her beloved adult stepson, established with a bank a passbook savings account by an initial deposit of $10,000. The passbook was issued solely in the stepson's name, but the widow retained possession of it, and the stepson was not then informed of the savings account. Subsequently, the widow became disgusted with the stepson's behavior and decided to give the same savings account solely to her beloved adult daughter. As permitted by the rules of the bank, the widow effected this change by agreement with the bank. This time she left possession of the passbook with the bank. Shortly thereafter, the stepson learned of the original savings account in his name and the subsequent switch to the widow's daughter's name. If the stepson now sues the bank for $10,000 plus accrued interest, will the action succeed?

A. Yes, because the stepson was a third-party intended beneficiary of the original widow-bank deposit agreement. B. Yes, because the stepson was a constructive assignee of the widow's claim, as depositor, to the savings account. C. No, because the stepson never obtained possession of the passbook. D. No, because the stepson's right, if any, to the funds on deposit was effectively abrogated by the second widow-bank deposit agreement. Answer: Answer D is correct. To the extent that the stepson was a third-party beneficiary of the deposit agreement, his rights were abrogated before they vested because he neither knew about nor relied upon the existence of the deposit account prior to abrogation. For this reason A is incorrect. B is incorrect because the widow did not assign her rights under another agreement to her stepson when she established the first deposit agreement. C is incorrect because the stepson's possession of the passbook was not necessary for the right to vest. His possession would constitute performance of the duty to him under the contract after the right had vested

A group of injured citizens brought suit against an oil company in federal district court claiming that the company violated tort law after an oil leak caused harm to the citizens' lands. The oil company moved to dismiss on the ground that the citizens' suit contained a political question, rendering the case non-justiciable in federal court. The federal court denied the motion, and the oil company then promptly filed an appeal. In their appeal, the oil company claimed that the federal appellate court had jurisdiction to determine whether there was a political question under the collateral order doctrine. Does the appellate court have jurisdiction over this issue under the collateral order doctrine?

A. Yes, because the trial court ruled conclusively on the disputed question. B. Yes, because the issue does not substantially overlap with the merits of the case. C. No, because the decision would be reviewable on appeal after a final judgment. D. No, because the oil company must wait thirty (30) days after a final judgment to appeal an issue. Answer: C is correct. Even though the trial court conclusively ruled on the motion, and the issue does not overlap with the merits of the tort claim, the appellate court would not have jurisdiction under the collateral order doctrine because this issue could be reviewed on a standard appeal after a final judgment. This doctrine is meant to be only a narrow exception that has only been applied to cases involving immunity and double jeopardy, rights that would effectively be lost if the party was forced to stand trial before being allowed an appeal. The collateral order doctrine is a narrow exception to the finality requirement for appeals. Typically only final order are reviewable by the appellate court, however under this doctrine a claim or issue may be immediately appealable if it is too important to wait. There are three requirements: (1) the lower court must have conclusively determined the disputed question; (2) the issue must be separate from and collateral to the merits of the main issue of the case; and (3) the issue must be effectively unreviewable on an appeal from the final judgment.

A federal government agency brought suit against a stockbroker in federal district court for fraud in violation of the federal securities laws. No jury trial was available so the suit was tried by a judge. It resulted in judgment against the stockbroker and imposition of a fine. Later, a man brought suit against the stockbroker to recover for the financial harm suffered from the very same fraud in violation of the federal securities laws. The man's lawsuit was an action at law to which the Seventh Amendment right to jury trial attached. The man sought to preclude the stockbroker from re-litigating the issues of fact that were the basis of the judgment entered against the stockbroker in the previous suit and that were identical to issues also presented by the man's claim. The stockbroker argued that, under the Seventh Amendment, he had a right to a jury trial on those factual issues. Should the court grant the man's motion to collaterally estop the stockbroker from re-litigating the factual issues that were decided against him in the previous case? You may assume that all of the requirements for offensive non-mutual collateral estoppel are met.

A. Yes, because when all of the requirements for offensive non-mutual collateral estoppel are met, there is no further fact-finding function for the jury to perform. B. No, because to collaterally estop the stockbroker from litigating the fact issues to a jury, when he did not have the opportunity to do so in the past, constitutes a deprivation of the stockbroker's Seventh Amendment right to jury trial. C. No, because federal courts do not accept the doctrine of offensive non-mutual collateral estoppel. D. Yes, because no Seventh Amendment right to jury trial attaches to statutory claims that did not exist when the Seventh Amendment was added to the Constitution. Answer: A. The court should grant the man's motion to collaterally estop the stockbroker from re-litigating the factual issues that were decided against the stockbroker in the government's case. When all of the requirements for offensive non-mutual collateral estoppel have been met, there is no further fact-finding function for the jury to perform. Therefore, denying the stockbroker a jury trial on those previously litigated issues would not violate the stockbroker's Seventh Amendment right to a jury trial.

Suspecting that a husband had slain his wife, police detectives persuaded one of the husband's colleagues at work to remove a drinking glass from the husband's office so that it could be used for fingerprint comparisons with a knife found near the body The fingerprints matched. The prosecutor announced that he would present comparisons and evidence to the grand jury. The husband's lawyer immediately filed a motion to suppress the evidence of the fingerprint comparisons so as to bar its consideration by the grand jury, contending that the evidence was illegally acquired. Should the motion be granted?

A. Yes, because, if there was no probable cause, the grand jury should not consider the evidence. B. Yes, because the employee was acting as a police agent and his seizure of the glass without a warrant was unconstitutional C. No, because motions based on the exclusionary rule are premature in grand jury proceedings. D. No, because the glass was removed from the husband's possession by a private citizen and not a police officer. Answer: C. The motion should be denied. The issue in this question is not whether the seizure of the glass was valid, but whether it is an appropriate time to raise this issue. The exclusionary rule does not apply in grand jury proceedings. According to US v. Calandra, illegally seized evidence is admissible in grand jury proceedings. A pretrial motion to suppress is the appropriate vehicle to test the constitutionality of the seizure. The grand jury is not the appropriate forum.

While walking on a public sidewalk, a pedestrian was struck by a piece of lumber that fell from the roof of a homeowner's house. The homeowner had hired a repairman to make repairs to his roof, and the lumber fell through due to negligence on the repairman's part. Assume that the homeowner exercised reasonable care in hiring the repairman, that the repairman was an independent contractor, and that public policy made a homeowner's duty to keep the sidewalk safe for pedestrian a nondelegable duty. If the pedestrian brings an action against the homeowner to recover damages for the injury caused to him by the repairman's negligence, will the pedestrian prevail?

A. Yes, under the res ipsa loquitur doctrine. B. Yes, because the repairman's act was a breach of a nondelegable duty owed by the homeowner to the pedestrian. C. No, because the repairman was an independent contractor rather than the homeowner's servant. D. No, because the homeowner exercised reasonable care in hiring the repairman to do the repair. Answer: B is the correct answer. In general, an independent contractor is liable for his own torts. However, an exception exists where the contractor is carrying out an inherently dangerous activity or where there is a public policy consideration that makes the duty nondelegable. The facts indicate that the duty to make the sidewalk safe for pedestrians was a nondelegable duty. Therefore, Answer B is the best choice. Answer A is inapplicable on the facts, which clearly state that the repairman was negligent. Res ipsa loquitur is only used where proof of negligence must be inferred. Answer C reaches the wrong conclusion. The homeowner will still be held liable for the actions of an independent contractor because he had a nondelegable duty. The word servant is a term of art and refers to a hired individual with duties defined and under the control of the one who did the hiring. Answer D is not the best answer. Even though the homeowner exercised reasonable care in hiring the repairman, he still had a nondelegable duty to keep the sidewalk safe.

While a woman was in her kitchen, she heard the screech of automobile tires. She ran to the window and saw a tricycle flying through the air. The tricycle had been hit by a car driven by a young man, who had been speeding. She also saw a child's body in the grass adjacent to the street. As a result of her shock from this experience, the woman suffered a heart attack. In a claim by the woman against the young man, the woman's right to recovery will depend on whether

A. a person can recover damages based on the defendant's breach of a duty owed to another. B. it is foreseeable that a person may suffer physical harm caused solely by an injury inflicted on another. C. a person can recover damages caused by shock unaccompanied by bodily impact. D. a person can recover damages for harm resulting from shock caused solely by another's peril or injury. Answer: D is the correct answer. Traditionally, there are two requirements for a negligent infliction of emotional distress claim. First, the plaintiff must suffer physical injury from the emotional distress. Second, the plaintiff must be within the zone of danger. However, courts will waive the zone of danger requirement so long as three factors are present: (1) the plaintiff and the person injured are closely related, (2) the plaintiff was present at the scene of the injury, and (3) the plaintiff personally perceived the event. Here, the facts clearly state that the woman had a physical injury due to the emotional distress, namely a heart attack. Therefore, as the woman was clearly outside of the zone of danger, her right to recovery will depend on whether she can recover emotional distress damages caused solely by another's peril or injury. Thus, answer D is the best choice even though the woman will be unable to satisfy the exception's requirements. Answers A and B are incorrect because they do not address the relevant requirements of a negligent infliction of emotional distress cause of action. Answer C is incorrect because in this case the shock clearly resulted in a physical injury.

A plaintiff sued a defendant for breach of a commercial contract in which the defendant had agreed to sell the plaintiff all of the plaintiff's requirements for widgets. The plaintiff called an expert witness to testify as to damages. On cross-examination, the defendant seeks to elicit from the expert witness that he had provided false testimony as a witness in his own divorce proceedings. This evidence should be

A. admitted, because the evidence was elicited from the expert witness on cross-examination. B. admitted, because the questioning can be substantiated by clear and convincing extrinsic evidence. C. excluded, because it is impeachment on a collateral issue. D. excluded, because it is improper character evidence. Answer: A. Specific instances of misconduct of a witness, for the purpose of attacking or supporting the witness's character for truthfulness, may not be proved by extrinsic evidence. They may, however, in the discretion of the court, if probative of truthfulness or untruthfulness, be inquired into on cross-examination of the witness. The expert witness's false testimony in his divorce proceeding is probative of his character for truthfulness or untruthfulness, and can be inquired into upon in cross-examination.

During 2006 a series of arsons, one of which damaged a huge store, occurred in a city. In early 2007 the city's City Council adopted this resolution: The City will pay $10,000 for the arrest and conviction of anyone guilty of any of the 2006 arsons committed here. The foregoing was telecast by the city's sole television station once daily for one week. Subsequently, the store, by a written memorandum to a private detective, proposed to pay the detective $200 "for each day's work you actually perform in investigating our fire." Thereafter in August, 2007, the City Council by resolution repealed its reward offer and caused this resolution to be broadcast once daily for a week over two local radio stations, the local television station having meanwhile ceased operations. In September, 2007, an employee of the store voluntarily confessed to the detective to having committed all of the 2006 arsons. The store's president thereupon paid the detective at the proposed daily rate for his investigation and suggested that the detective also claim the city's reward, of which the detective had been previously unaware. The detective immediately made the claim. In December, 2007, as a result of the detective's investigation, the store's employee was convicted of burning the store. The city, which has no immunity to suit, has since refused to pay the detective anything, although he swears that he never heard of the city's repealer before claiming its reward. If the city's reward offer was revocable, revocation could be effectively accomplished only

A. by publication in the legal notices of a local newspaper. B. in the same manner as made, i.e., by local telecast at least once daily for one week. C. in the same manner as made or by a comparable medium and frequency of publicity. D. by notice mailed to all residents of the city and all other reasonably identifiable, potential offerees. Answer: C. Revocation of offers made to the general public must be given publicity equal to that given to the offer and no better means of notification should be reasonably available. A is incorrect because revocation could be completed by any medium comparable to television as long as it was made with similar frequency. B is incorrect because the television station is no longer available and the rules of revocation allow notification by other means reasonably available. D is incorrect because revocation of an offer to the general public does not require actual notice.

The legislature of a particular state enacted a statute requiring that all law enforcement officers in that state be citizens of the United States. An alien, lawfully admitted to permanent residency five years before the enactment of this statute, sought employment as a forensic pathologist in the state corner's office. He was denied such a job solely because he was not a citizen. The alien thereupon brought suit in federal district court against appropriate state officials seeking to invalidate this citizenship requirement on federal constitutional grounds. The strongest ground upon which to attack this citizenship requirement is that is

A. constitutes an ex post facto law as to previously admitted aliens. B. deprives an alien of a fundamental right to employment without the due process of law guaranteed by the Fourteenth Amendment C. denies an alien a right to employment in violation of the privileges and immunities clause of the Fourteenth Amendment. D. denies an alien the equal protection of the laws guaranteed by the Fourteenth Amendment. Answers: D. The Equal Clause requires that classifications based on alienage be narrowly tailored to promote a compelling state interest. Because that is such a difficult test to satisfy, this is the strongest argument.

In order to provide funds for a system of new major airports near the ten largest cities in the United States, Congress levies a tax of $25 on each airline ticket issued in the United States. The tax applies to every airline ticket, even those for travel that does not originate in, terminate at, or pass through any of those ten large cities. As applied to the issuance in the United States of an airline ticket for travel between two cities that will not be served by any of the new airports, this tax is

A. constitutional, because Congress has broad discretion in choosing the subjects of its taxation and may impose taxes on subject that have no relation to the purpose for which those tax funds will be expended. B. constitutional, becuase an exemption for the issuance of tickets for travel between cities that will not be served by teh new airports would deny the purchasers of all other tickets the equal protection of the laws. C. unconstitutional, because the burden of the tax outweighs its benefits for passengers whose travel does not originate in, terminate at, or pass through any of the ten largest cities. D. unconstitutional, because the tax adversely affects the fundamental right to travel. Answer: A. Because Congress's powers to tax and to spend funds are both very broad, and there is no requirement that what is spent be related in any way to what is taxed. Answer B is incorrect because an exemption for travelers between other cities would not deny any purchasers equal protection of the law, since such an exemption would be rationally related to the legitimate governmental purpose of funding the busier airports. Answer C is incorrect because the test is not whether the burden outweighs the benefits for the travelers. In fact, the burdens and benefits are only relevant if they make the taxing irrational. Answer D is incorrect because the tax does not infringe on the right to travel, since it applies equally to all travel, is a small part of the total cost of place fare, and there are many other easy ways to travel between the same destinations.

A major corporation is privately owned and incorporated in State A. It contracted with the United States to construct a dam across a river in State B. State B imposed a gross receipts tax on all business conducted within the state. State B sued the corporation to collect that tax on the receipts the corporation received under this federal contract. No federal statutes or administrative rules are applicable, and the contract between the United States and the corporation does not mention state taxation. The court should hold the state tax, as applied here, to be

A. constitutional, because a state has exclusive jurisdiction over all commercial transactions executed wholly within its borders. B. constitutional, because private contractors performing work under a federal contract are not immune in these circumstances from nondiscriminatory state taxation. C. unconstitutional, because it violates the supremacy clause. D. unconstitutional, because it imposes an undue burden on interstate commerce.

Congress passed a bill prohibiting the President from granting a pardon to any person who had not served at least one-third of the sentence imposed by the court which convicted that person. The President vetoed the bill, claiming that it was unconstitutional. Nevertheless, Congress passed it over his veto by a two-thirds vote of each house. This act of Congress is

A. constitutional, because it was enacted over the President's veto by a two-thirds vote of each house. B. constitutional, because it is a necessary and proper means of carrying out the powers of Congress. C. unconstitutional, because it interferes with the plenary power of the President to grant pardons. D. unconstitutional, because a Presidential veto based upon constitutional grounds may be overridden only with the concurrence of three-fourths of the state legislatures. Answer: C. Although Congress can usually enact laws that the President has vetoed by a two-thirds majority vote (as choice A would suggest), Congress cannot take power away from the President that the Constitution grants. Article II, § 2, cl. 1 states that the President shall have power to grant pardons except in cases of impeachment. That power is not qualified in any way, and for Congress to attempt to qualify the power is unconstitutional. B is incorrect because the power to grant pardons is not a Congressional power; rather it is solely an executive power. D is incorrect because there is no such rule for vetoes on unconstitutional grounds. The Constitution can be amended when a proposed amendment, passed by two-thirds of both houses of Congress, is ratified by three-fourths of the states, but that was not the scenario in this problem.

A motorist arranged to borrow his friend's car to drive for one day while the motorist's car was being repaired. The friend knew that the brakes on his car were faulty and might fail in an emergency. The friend forgot to tell the motorist about the brakes when the motorist picked up the car, but the friend did telephone the motorist's wife and told her about them. The wife, however, forgot to tell the motorist. The motorist was driving the friend's car at a reasonable rate of speed and within the posted speed limit with the motorist's wife as a passenger. Another car, driven by a woman, crossed in front of the motorist at an intersection and in violation of the traffic signal. The motorist tried to stop, but the brakes failed, and the two cars collided. If the brakes had been in proper working order, the motorist could have stopped in time to avoid the collision. The motorist and his wife were injured. If the jurisdiction has adopted "pure" comparative negligence and the motorist's wife asserts a claim against the woman, the wife will

A. recover in full for her injuries, because the motorist, who was driving the car in which she was riding, was not himself at fault. B. recover a proportion of her damages based on the respective degrees of her negligence and that of the woman. C. not recover, because but for the failure of the brakes the collision would not have occurred. D. not recover, because she was negligent and her negligence continued until the moment of impact. Answer: B correctly states the appropriate standard, which will allow the motorist's wife to recover from the woman, proportionate to her percentage of fault, because the wife's negligence in failing to tell the motorist about the faulty brakes likely contributed to the collision - if the motorist had known, he may have swerved or driven slower. Even if the wife is 90% at fault and the woman only 10% at fault, the wife will recover 10% of her damages under the "pure" comparative negligence standard. A is incorrect. Under modern law, the wife cannot impute the motorist's liability status onto herself, or vice versa. C is incorrect. The woman had a duty to obey the traffic lights, and it was foreseeable that breaching this duty and crossing in violation of a light would result in harm to an opposing motorist; the brake failure of the motorist's car does not absolve the woman of liability. The woman's action is the legal cause of the collision. D incorrectly states the "pure" comparative negligence standard. Instead, it describes the traditional contributory negligence bar, which is only still used in a small minority of jurisdictions.

As part of a comprehensive federal aid-to-education program, Congress included the following provisions as conditions for state receipt of federal funds: (1) Whenever textbooks are provided to students without charge, they must include no religious instruction and must be made available on the same terms to students in all public and private schools accredited by the state educational authority. (2) Salary supplements can be paid to teachers in public and private schools, up to ten percent of existing salary schedules, where present compensation is less than the average salary for persons of comparable training and experience, provided that no such supplement is paid to any teacher who instructs in religious subjects. (3) Construction grants can be made toward the cost of physical plant at private colleges and universities, provided that no part of the grant is used for buildings in which instruction in religious subject matters is offered. A federal taxpayer challenges the salary supplements for teachers in private schools where religious instruction is included in the curriculum. On the substantive constitutional issue, the most likely result is that the salary supplements will be

A. sustained, because the statute provides that no supplements will be made to teachers who are engaged in any religious instruction. B. sustained, because to distinguish between private and public school teachers would violate the religious freedom clause of the First Amendment. C. held unconstitutional, because some religions would benefit disproportionately. D. held unconstitutional, because the policing of the restriction would amount to an excessive entanglement with religion. Answer: D is the correct answer. The statute stipulates the supplements are not to be paid to a teacher who instructs in religious subjects. To enforce this restriction, the state would have to monitor the curriculum of each teacher receiving a supplement to ensure he or she was not instructing on a religious subject. Thus, enforcement would create an excessive entanglement with religion and violate the Establishment Clause. This question is based on actual supreme court precedent where the court struck down programs paying a portion of private school teachers' salaries for their secular classes, since the primary effect would create a system that would need to be constantly monitored to ensure that the aid was not being used by the teachers for religious purposes. This constant monitoring would be an excessive entanglement, and therefore would not pass the third prong of the Lemon Test. (See, Grand Rapids School District v. Ball, 473 U.S. 373 (1985); Lemon v. Kurtzman, 403 U.S. 602 (1971)).

A realty company developed a residential development which included a single-family dwellings, town houses, and high-rise apartments for a total of 25,000 dwelling units. Included in the deed to each unit was a covenant under which the grantee's "heirs and assigns" agreed to purchase electrical power only from a plant the realty company promised to build and maintain within the development. The realty company constructed the plant and necessary power lines. The plant did not supply power outside the development. An appropriate and fair formula was used to determine price. After constructing and selling 12,500 of the units, the realty company sold its interest in the development to an investor company. The investor company operated the power plant and constructed and sold the remaining 12,500 units. Each conveyance from the investor company contained the same covenant relating to electrical power that the realty company had included in the 12,500 conveyances it had made. A woman bought a dwelling unit from a man, who had purchased it from the realty company. Subsequently, the woman, whose lot was along the boundary of the development, ceased buying electrical power from the investor company and began purchasing power from General Power Company, which provided such service in the area surrounding the development. Both General Power and the investor company have governmental authorization to provide electrical services to the area. The investor company instituted an appropriate action against the woman to enjoin her from obtaining electrical power from General Power. If judgment is for the woman, it will most likely be because

A. the covenant does not touch and concern the land. B. the mixture of types of residential units is viewed as preventing one common development scheme. C. the covenant is a restraint on alienation. D. there is no privity of estate between the woman and the investor company. Answer: A. The realty company has attempted to impose an equitable servitude on the residents of the subdivision. An equitable servitude in a deed is only enforceable where a party can establish: (1) intent for the restriction to be enforceable by subsequent grantees, (2) that the subsequent grantees had notice of the servitude, and (3) that the restriction touches and concerns the land. In this case, the facts clearly indicate that there was an intent for the restriction to be enforceable, and that the language of the deed put subsequent grantees on notice. The restriction, however, does not touch and concern the land; it does not serve to make the land more useful or valuable in any way. Therefore, the restriction will not bind the woman/

The owner of Blackacre, a single-family residence, conveyed a life estate in Blackacre to a landlord fifteen years ago. Fourteen years ago, the landlord, who had taken possession of Blackacre, leased Blackacre to a tenant for a term of 15 years at the monthly rent of $500. Eleven years ago, the landlord died intestate leaving her son as her sole heir. The tenant regularly paid rent to the landlord and, after the landlord's death, to the son until last month. The period in which to acquire title by adverse possession in the jurisdiction is 10 years. In an appropriate action, the tenant, the original owner, and the son each asserted ownership of Blackacre. The court should hold that title in fee simple is in?

A. the original owner, because the owner held a reversion and the landlord has died. B. the son, because the landlord asserted a claim adverse to the owner when the landlord executed a lease to the tenant. C. the son, because the tenant's occupation was attributable to the son, and the landlord died 11 years ago. D. the tenant, because of the tenant's physical occupancy and because the tenant's term ended with the landlord's death. Answer: C is correct. The interest that the original owner conveyed to the landlord was a life estate. As such, any right the landlord (or her assigns) had in the property ended upon her death. The son, having no legal right to the owner's property after the landlord's death, collected rent on that property from the tenant for eleven years. The son's actions have earned him an ownership interest in the property through adverse possession. To obtain land through adverse possession, a party must for the length of the statutory period: 1) have actual physical possession or occupancy of the land, 2) maintain that possession continuously and without interruption, 3) exclude others from possession, 4) have "hostile" possession (be there without permission), and 5) maintain "open and notorious" possession. In this case, the son held the property without permission, did so exclusively, openly and notoriously (by allowing his tenant but no other parties to physically possess that land and pay him rent), and did so without interruption for the length of the statutory period (which in this jurisdiction is ten years).

A man's estate plan included a revocable trust established 35 years ago with a bank as trustee. The principal asset of the trust has always been Blackacre, a very profitable, debt-free office building. The trust instrument instructs the trustee to pay the net income to the man for life, and, after the death of the man, to pay the net income to his wife for life; and, after her death, to distribute the net trust estate as she may appoint by will, or in default of her exercise of this power of appointment, to the man's son (her stepson). The man died 30 years ago survived by his wife and son. The man had not revoked or amended the trust agreement. A few years after the man's death, his wife remarried. She then had a daughter, was widowed for a second time, and then died last year. Her will contained only one dispositive provision: "I give my entire estate to my daughter, and I intentionally make no provision for my stepson." The daughter is now 22 years old. The common-law Rule Against Perpetuities is unmodified by statute in the jurisdiction. There are no other applicable statutes. The stepson brought an appropriate action against the daughter to determine who was entitled to the net trust estate and thus to Blackacre. If the court rules for the daughter, it will be because

A. the wife's life estate and general power of appointment merge into complete ownership in the wife. B. the Rule Against Perpetuities does not apply to general powers of appointment. C. the jurisdiction deems "entire estate" to be a reference to Blackacre or to the wife's general power of appointment. D. the wife intended that her stepson should not benefit by reason of her death. Answer: C. The fact pattern states that the wife was given a general power of testamentary "appointment" over the net trust estate of Blackacre. The use of that terminology created a specific power that could only be utilized in the wife's will. The issue is whether the wife appropriately exercised that power. Most jurisdictions require the language of the will to specifically refer to the exercise of the power, which the wife's does not do. Therefore, for the daughter to prevail, the court must find that the wife's general reference to her "entire estate" was a use of her power of appointment.


संबंधित स्टडी सेट्स

LJ Form A, LJ Form G, LJ Form M, LJ Form E, LJ Form I, LJ form K, LJ form B, LJ FORM F, LJ FORM H, LJ FORM D, LJ FORM L, LJ FORM J

View Set

software engineering collection 5

View Set

define each of the types of constitutional powers

View Set